You are on page 1of 80

Volume 24 No.

3 March 2016
Managing Editor Corporate Office:
Mahabir Singh Plot 99, Sector 44 Institutional area, Gurgaon -122 003 (HR).
Editor Tel : 0124-4951200 e-mail : info@mtg.in website : www.mtg.in
Anil Ahlawat Regd. Office:
(BE, MBA) 406, Taj Apartment, Near Safdarjung Hospital, New Delhi - 110029.

Physics Musing Problem Set 32 8

CONTENTS AIPMT Practice Paper


Core Concept
12
20
JEE Main Practice Paper 24

JEE Accelerated Learning Series 31


Brain Map 46
Exam Prep 2016 60
JEE Advanced Practice Paper 67
AIPMT Model Test Paper 2016 72
Physics Musing Solution Set 31 85
Live Physics 87
You Ask We Answer 88
Crossword 89

subscribe online at www.mtg.in


individual subscription rates combined subscription rates
3 yrs.
1900
1900
2300
Biology Today 330 600 775
Send D.D/M.O in favour of MTG Learning Media (P) Ltd.
Payments should be made directly to : MTG Learning Media (P) Ltd,
Plot No. 99, Sector 44, Gurgaon - 122003 (Haryana)
We have not appointed any subscription agent.
Owned, Printed and Published by Mahabir Singh from 406, Taj Apartment, New Delhi - 29 and printed
by Personal Graphics and Advertisers (P) Ltd., Okhla Industrial Area, Phase-II, New Delhi. Readers are
adviced to make appropriate thorough enquiries before acting upon any advertisements published in this
magazine. Focus/Infocus features are marketing incentives. MTG does not vouch or subscribe to the claims
and representations made by advertisers. All disputes are subject to Delhi jurisdiction only.
Editor : Anil Ahlawat
Copyright MTG Learning Media (P) Ltd.
All rights reserved. Reproduction in any form is prohibited.

Physics for you | March 16 7


PHYSICS MUSING
P hysics Musing was started in August 2013 issue of Physics For You with the suggestion of Shri Mahabir Singh. The aim of Physics Musing is to augment
the chances of bright students preparing for JEE (Main and Advanced) / AIIMS / Other PMTs with additional study material.
In every issue of Physics For You, 10 challenging problems are proposed in various topics of JEE (Main and Advanced) / various PMTs. The detailed
solutions of these problems will be published in next issue of Physics For You.
The readers who have solved five or more problems may send their detailed solutions with their names and complete address. The names of those who
send atleast five correct solutions will be published in the next issue.
We hope that our readers will enrich their problem solving skills through Physics Musing and stand in better stead while facing the competitive exams.

single oPtion correct tyPe gravitationally. Find their velocity of approach


when they are separated by a distance s.
1. Find the inductance of a unit length of two long
parallel wires, each of radius a, whose centers are a (a) G ( M1 + M2 ) (b) GM1M2
distance d apart and carry equal currents in opposite 2s s
directions. Neglect the flux within the wire. Gs
(c) 2G ( M1 + M2 ) (d)
d a
(a) 0 ln d a (b) 0 ln s
M1M2
2 a a
5. A system S receives heat continuously from an
30 d a 0 d a electrical heater of power 10 W. The temperature of
(c) ln (d) ln
a 3 a S becomes constant at 50C when the surrounding
2. From a cylinder of radius R, a temperature is 20C. After the heater is switched
cylinder of radius R/2 is removed, off, S cools from 35.1C to 34.9C in 1 minute. The
as shown in the figure. Current heat capacity of S is
flowing in the remaining cylinder is (a) 100 JC1 (b) 300 JC1
(c) 750 JC 1 (d) 1500 JC1
I. Then, magnetic field strength is
(a) zero at point A 6. A flywheel rotating about an axis experiences
(b) zero at point B an angular retardation proportional to the angle
I through which it rotates. If its rotational kinetic
(c) 0 at point A
2R energy gets reduced by E while it rotates through
I an angle , then
(d) 0 at point B.
3R (a) E 2 (b) E
3. A beam of the light is incident vertically on a glass (c) E (d) E 3/2
hemisphere of radius R and refractive index 2 , comPrehension tyPe
lying with its plane side on a table. The axis of beam For questions 7 and 8
coincides with the vertical axis passing through the The value of potential energy at the reference point itself
centre of base of the hemisphere and cross sectional can be set equal to zero because we are always concerned
radius of beam is R . The luminous spot formed only with differences of potential energy between two
2 points and the associated change of kinetic energy. A
on the table is of radius particle A is fixed at origin of a fixed coordinate system.
R Another particle B which is free to move experiences a
(a) R (b) 2
2
force F = + r due to particle A where r is the
3 2
2R r r
(c) (d) ( 3 + 1) R
( 3 + 1) position vector of the particle B relative to A. It is given
that the force is conservative in nature and potential
4. Two masses M1 and M2 at an infinite distance energy at infinity is zero. If it has to be removed from
apart are initially at rest. They start interacting the influence of A, energy has to be supplied for such a

8 Physics for you | march 16


process. The ionization energy E0 is the work
that has to be done by an external agent to
move the particle from a distance r0 to infinity
slowly. Here r0 is the equilibrium position of
the particle.
7. What is the potential energy function of
particle as a function of r?

(a) (b) +
2 r
r r2 r

(c) (d) +
2 r
r r2 r
8. Find the ionization energy E0 of the particle
B.
2 22
(a) (b)
2
2 2
(c) (d)
4
For questions 9 and 10
A parallel plate capacitor is filled with dielectric
material. If the capacitor is charged, electric field
is created inside the dielectric. Due to this field,
the electrons (which are not free), experience
force in opposite direction of the field. If a very
high field is applied in the dielectric, the outer
electrons may get detached from the atoms and
then the dielectric behaves like a conductor. This
phenomenon is called dielectric breakdown.
The minimum field at which the breakdown
occurs is called the dielectric strength of the
material and corresponding potential is called
breakdown potential.
There are two capacitors of capacitances C and
2C. The breakdown potential of each capacitor
is V0.
9. If they are joined in series, then the
maximum potential difference that can be
applied across the combination for their
safely use will be
3
(a) V0 (b) V0
2
(c) 2V0 (d) 3V0
10. If the voltage across the parallel combination
of these two capacitors is increased, which
capacitor will undergo breakdown first?
(a) C
(b) 2C
(c) Both at same moment
(d) None of these
nn
Courtesy : The Times of India
10 Physics for you | march 16
*K P Singh

set 1 rotational motion | Gravitation | mechanical ProPerties of solids and fluids

1. The ratio of the radii of gyration of a circular disc 6. The excess pressure inside a spherical drop of radius
about a tangential axis in the plane of the disc and of r of a liquid of surface tension T is
a circular ring of the same radius about a tangential (a) directly proportional to r and inversely
axis in the plane of the ring is proportional to T
(a) 3 : 5 (b) 12 : 3 (b) directly proportional to T and inversely
proportional to r
(c) 1 : 3 (d) 5: 6 (c) directly proportional to the product of T and r
2. Three identical bodies of mass M are located at the (d) inversely proportional to the product of T and r.
vertices of an equilateral triangle of side L. They
7. A piece of ice is floating in a jar containing water.
revolve under the effect of mutual gravitational
When the ice melts, then the level of water
force in a circular orbit, circumscribing the triangle
(a) rises
while preserving the equilateral triangle. Their
(b) falls
orbital velocity is
(c) remains unchanged
GM 3 GM 3GM 2 GM (d) either rises or falls.
(a) (b) (c) (d)
L 2L L 3L 8. The average depth of Indian ocean is about 3000 m.
3. A stone of mass m tied to a string of length L is V
rotating along a circular path with constant speed The fractional compression, of water at the
V
v. The torque on the stone is bottom of the ocean (given that the bulk modulus
mv mv 2 of the water = 2.2 109 N m2 and g = 10 m s2) is
(a) mLv (b) (c) (d) zero (a) 0.82 % (b) 0.91 % (c) 1.36 % (d) 1.24 %
L L
4. A body A of mass M while falling vertically 9. If linear density of a rod of length 3 m varies as
downwards under gravity breaks into two parts, a l = 2 + x, then the position of the centre of gravity
1 2 of the rod is
body B of mass M and a body C of mass M .
3 3 7 12 10 9
The centre of mass of bodies B and C taken together (a) m (b) m (c) m (d) m
3 7 7 7
as compared to centre of mass of body A, 10. A uniform rod of length 8a and mass 6m lies on
(a) shifts depending on height of breaking a smooth horizontal surface. Two point masses m
(b) does not shift and 2m moving in the same plane with speed 2v
(c) shifts towards body C and v respectively strike the rod perpendicularly at
(d) shifts towards body B distances a and 2a from the mid point of the rod
5. A 20 cm long capillary tube is dipped in water. The in the opposite directions and stick to the rod. The
water rises upto 8 cm. If the entire arrangement is angular velocity of the system immediately after the
put in a freely falling elevator, the length of water collision is
column in the capillary tube will be 6v 6v 6v 6v
(a) 8 cm (b) 10 cm (c) 4 cm (d) 20 cm (a) (b) (c) (d)
32 a 33 a 40 a 41 a
*A renowned physics expert, KP Institute of Physics, Chandigarh, 09872662552

12 Physics for you | march 16


11. Four wires of the same material are stretched by the (a) (r1 + r2) (b) (r1 + r2)1/2
same load. Which one of them will elongate most if (c) (r1 r2)3/2 (d) (r1 + r2)3/2
their dimensions are as follows ?
19. The surface tension of soap solution is 0.03 N m1.
(a) L = 100 cm, r = 1 mm
The work done (in J) in blowing to form a soap
(b) L = 200 cm, r = 3 mm
(c) L = 300 cm, r = 3 mm bubble of surface area 40 cm2, is
(d) L = 400 cm, r = 4 mm (a) 1.2 104 (b) 2.4 104
4
(c) 12 10 (d) 24 104
12. The cylindrical tube of a spray pump has a
cross-section of 8 cm2, one end of which has 40 fine 20. Three capillaries of lengths L, L/2 and L/3 are
holes each of area 108 m2. If the liquid flows inside connected in series. Their radii are r, r/2 and r/3
the tube with a speed of 0.15 m min1, the speed respectively. Then, if stream-line flow is to be
with which the liquid is ejected through the holes is maintained and the pressure across the first capillary
(a) 50 m s1 (b) 5 m s1 is p, then the
1
(c) 0.05 m s (d) 0.5 m s1 (a) pressure difference across the ends of second
13. Two particles of equal mass have velocities capillary is 8 p
(b) pressure difference across the third capillary is
v1 = 4 i m s 1 and v2 = 4 j m s 1. First particle has
43 p
an acceleration a1 = (5 i + 5 j) m s 2 , while the (c) pressure difference across the ends of the second
acceleration of the other particle is zero. The centre capillary is 16 p
of mass of the two particles moves in a path of (d) pressure difference across the third capillary is
(a) straight line (b) parabola
56 p.
(c) circle (d) ellipse
21. The moment of inertia of a thin circular disc about
14. The change in potential energy when a body of mass
an axis passing through its centre and perpendicular
m is raised to a height nR from earths surface is
to its plane is I. Then, the moment of inertia of
(R = radius of the earth)
the disc about an axis parallel to its diameter and
n
(a) mgR (b) mgR touching the edge of the rim is
(n 1)
n n2 3 5
(c) mgR (d) mgR (a) I (b) 2 I (c) I (d) I
(n + 1) (n2 + 1) 2 2
22. In an elliptical orbit under gravitational force, in
15. Two drops of equal radius coalesce to form a bigger
general
drop. What is ratio of surface energy of bigger drop
(a) tangential velocity is constant
to smaller one?
(a) 21/2 : 1 (b) 1 : 1 (b) angular velocity is constant
(c) 22/3 : 1 (d) None of these (c) radial velocity is constant
(d) areal velocity is constant.
16. Two capillaries of lengths L and 2L and of radii R
and 2R are connected in series. The net rate of flow 23. A layer of glycerine of thickness 1 mm is present
of fluid through them will be (given rate of the flow between a large surface area and a surface area of
pR 4 0.1 m2. With what force the small surface is to be
through single capillary, X = ) pulled, so that it can move with a velocity of 1 m s1 ?
8 L
9 5 7 (Given that coefficient of viscosity = 0.07 kg m1 s1)
8
(a) X (b) X (c) X (d) 5 X (a) 70 N (b) 7 N
9 8 7
17. The angle turned by a body undergoing circular (c) 700 N (d) 0.70 N
motion depends on time as q = q0 + q1 t + q2t2. 2
Then the angular acceleration of the body is 24. The ratio of radii of earth to another planet is
3
(a) q1 (b) q2 (c) 2 q1 (d) 2 q2 4
and the ratio of their mean densities is . If an
18. A planet of mass m moves around the sun of 5
mass M in an elliptical orbit. The maximum and astronaut can jump to a maximum height of 1.5 m
minimum distances of the planet from the sun are on the earth, with the same effort, the maximum
r1 and r2 respectively. The time period of the planet height he can jump on the planet is
is proportional to (a) 1 m (b) 0.8 m (c) 0.5 m (d) 1.25 m

14 Physics for you | march 16


25. Two wires of same material and radius have their 32. Two rain drops reach the earth with different
lengths in ratio 1 : 2. If these wires are stretched by terminal velocities having ratio 9 : 4. Then the ratio
the same force, the strain produced in the two wires of their volumes is
will be in the ratio (a) 3 : 2 (b) 4 : 9 (c) 9 : 4 (d) 27 : 8
(a) 2 : 1 (b) 1 : 1 (c) 1 : 2 (d) 1 : 4 33. A door 1.6 m wide requires a force of 1 N to be
26. When the temperature increases, the viscosity of applied at the free end to open or close it. The force
(a) gas decreases and liquid increases that is required at a point 0.4 m distance from the
(b) gas increases and liquid decreases hinges for opening or closing the door is
(c) gas and liquid increase (a) 1.2 N (b) 3.6 N (c) 2.4 N (d) 4 N
(d) gas and liquid decrease. 34. A body is released from a point, distant r from the
27. A thin uniform square lamina of side a is placed in centre of earth. If R is the radius of the earth and
the xy-plane with its sides parallel to x and y-axis r > R, then the velocity of the body at the time of
and with its centre coinciding with origin. Its striking the earth will be
moment of inertia about an axis passing through a (a) gR (b) 2 gR
point on the y-axis at a distance y = 2a and parallel 2 gR 2 gR (r R)
to x-axis is equal to its moment of inertia about (c) (d)
an axis passing through a point on the x-axis at a rR r
distance x = d and perpendicular to xy-plane. Then 35. A wire of natural length L, Youngs modulus Y and
value of d is area of cross-section A is extended by x. Then the
7 47 9 51 energy stored in the wire is given by
(a) a (b) a (c) a (d) a 1 YA 2 1 YA 2
3 12 5 12 (a) x (b) x
2 L 3 L
28. Gravitational acceleration on the surface of a planet
6 1 YL 2 1 YA 2
g , where g is the gravitational acceleration on (c) x (d) x
is 2 A 2 L2
11
the surface of the earth. The average mass density of 36. Two spherical soap bubbles of radii r1 and r2 in
the planet is 2/3 times that of the earth. If the escape vacuum combine under isothermal conditions. The
speed on the surface of the earth is taken to be resulting bubble has a radius equal to
11 km s1, the escape speed on the surface of the r +r r1 r2
planet in km s1 will be (a) 1 2 (b) r + r
2 1 2
(a) 5 (b) 7 (c) 3 (d) 11
(c) r1 r2 (d) r12 + r22
29. An open U-tube contains mercury. When 11.2 cm
of water is poured into one of the arms of the tube, 37. A thin circular ring of mass M and radius R rotates
how high does the mercury rise in the other arm about an axis through its centre and perpendicular
from its initial unit? to its plane, with a constant angular velocity w. Four
(a) 0.56 cm (b) 1.35 cm small spheres each of mass m (negligible radius) are
(c) 0.41 cm (d) 2.32 cm kept gently to the opposite ends of two mutually
perpendicular diameters of the ring. The new
30. A manometer connected to a closed tap reads angular velocity of the ring will be
3.5 105 N m2. When the valve is opened, the M
reading of manometer falls to 3.0 105 N m2, then (a) 4 w (b) w
4m
velocity of flow of water is
(a) 100 m s1 (b) 10 m s1 M + 4m M
(c) w (d) w
(c) 1 m s1 (d) 10 10 m s1 M M + 4m
31. A rope 1 cm in diameter breaks, if the tension in it 38. If r is the density of the planet, the time period of
exceeds 500 N. The maximum tension that may be nearby satellite is given by
given to similar rope of diameter 3 cm is 4 4 3
(a) 500 N (b) 3000 N (a) (b) (c) (d)
3 Gr Gr Gr Gr
(c) 4500 N (d) 2000 N

Physics for you | march 16 15


39. Eight equal drops of water are falling through air 42. A body is orbiting very close to the earths surface
with a steady velocity of 10 cm s1. If the drops with kinetic energy KE. The energy required to
combine to form a single drop big in size, then the completely escape from it is
terminal velocity of this big drop is KE 3 KE
(a) 80 cm s1 (b) 30 cm s1 (a) KE (b) 2 KE (c) (d)
2 2
(c) 10 cm s1 (d) 40 cm s1 43. Three particles each of mass m are kept at vertices
40. An annular ring with inner and outer radii of an equilateral triangle of side L. The gravitational
R1 and R2 is rolling without slipping with a uniform field at centre due to these particles is
angular speed. The ratio of the forces experienced 3 GM 12 GM
9 GM
by the two particles situated on the inner and outer (a) zero (b) (c) (d)
F L2
L 2 3 L2
parts of the ring, i.e., 1 is 44. A body weighs 50 g in air and 40 g in water. How much
F2 2
R1 R1 R would it weigh in a liquid of specific gravity 1.5?
(a) (b) 1 (c) (d) 2 (a) 30 g (b) 35 g (c) 65 g (d) 45 g
R2 R2 R1
41. The potential energy of 4 particles each of mass 1 kg 45. When a number of small droplets combine to form
a large drop, then
placed at the four vertices of a square of side length (a) total volume decreases
1 m is (in SI units) (b) thermal energy increases
(a) + 4.0 G (b) 7.5 G (c) thermal energy decreases
(c) 5.4 G (d) + 6.3 G (d) surface energy increases.

set 2 oPtics | modern Physics | semiconductor electronics

1. A ray of light passes from vacuum into a medium (c) when they have constant phase difference and
of refractive index m, the angle of incidence is found different frequencies
to be twice the angle of refraction. Then angle of (d) when they have varying phase difference and
incidence is different frequencies.
1 m 1 m 5. If the energy of the photon is increased by a factor
(a) cos (b) 2 cos
2 2 of 4, then its momentum
(a) does not change
1 m
(c) 2 sin1 (m) (d) 2 sin (b) decreases by a factor of 4
2 (c) increases by a factor of 4
2. Consider the nuclear reaction X A110 + B80. If
200
(d) decreases by a factor of 2
the binding energy per nucleon for X, A and B are
6. u1 is the frequency of the series limit of Lyman
7.4 MeV, 8.2 MeV and 8.1 MeV respectively, then
series, u2 is the frequency of the first line of Lyman
the energy released in the reaction is
series and u3 is the frequency of the series limit of
(a) 70 MeV (b) 200 MeV
the Balmer series. Then,
(c) 190 MeV (d) 10 MeV
(a) u1 u2 = u3 (b) u1 = u2 u3
3. A ray of light passes through an equilateral prism
such that the angle of incidence is equal to the angle 1 1 1 1 1 1
of emergence and the latter is equal to 3/4 the angle (c) = + (d) = +
u2 u1 u3 u1 u2 u3
of prism. The angle of deviation is
(a) 25 (b) 30 7. A uniform electric field and a uniform magnetic
(c) 45 (d) 35 field exist in a region in the same direction. An
4. Two light sources are said to be coherent electron is projected with a velocity pointed in the
(a) when they have same frequency and a varying same direction. Then the electron will
phase difference (a) be deflected to the left without increase in speed
(b) when they have same frequency and a constant (b) be deflected to the right without increase in
phase difference speed

16 Physics for you | march 16


(c) not be deflected but its speed will decrease
(a) 3 : 2 (b) 2 : 1 (c) 2 : 1 (d) 4 : 1
(d) not be deflected but its speed will increase.
16. A prism having refractive index 1.414 and refracting
8. The current in the circuit shown in the figure
angle 30 has one of the refracting surfaces silvered. A
considering ideal diode is
beam of light incident on the other refracting surface
(a) 20 A
(b) 2 103 A will retrace its path, if the angle of incidence is
(c) 200 A (a) 45 (b) 60 (c) 30 (d) 0
(d) 2 104 A 17. The focal lengths of the objective and the eye piece
9. Wavelengths of light used in an optical instrument of telescope are 100 cm and 10 cm respectively. The
are l1 = 4000 and l2 = 5000 , then ratio of their magnification of the telescope when final image is
respective resolving powers (corresponding to l1 formed at infinity is
and l2) is (a) 0.1 (b) 10 (c) 100 (d)
(a) 16 : 25 (b) 9 : 1 (c) 4 : 5 (d) 5 : 4 18. If the kinetic energy of a free electron doubles, its
10. Which of the following is correct, about doping in a de Broglie wavelength changes by the factor
transistor? 1 1
(a) Emitter is lightly doped, collector is heavily (a) 2 (b) (c) 2 (d)
2 2
doped and base is moderately doped.
(b) Emitter is lightly doped, collector is moderately 19. The maximum efficiency of full wave rectifier is
doped and base is heavily doped. (a) 100 % (b) 25.2 % (c) 40.6 % (d) 81.2 %
(c) Emitter is heavily doped, collector is lightly 20. When an unpolarized light of intensity I0 is incident
doped and base is moderately doped. on a polarizing sheet, the intensity of the light which
(d) Emitter is heavily doped, collector is moderately does not get transmitted is
doped and base is lightly doped. I I
11. The temperature at which protons in proton gas (a) 0 (b) 0 (c) zero (d) I0
2 4
would have enough energy to overcome Coulomb 4 9 1
21. 2 He + 4 Be 0 n +?
barrier of 4.14 1014 J is
(Boltzmann constant = 1.38 1023 J K1) The missing ion in the given nuclear reaction is
(a) 2 109 K (b) 109 K (a) proton (b) oxygen-12
9
(c) 6 10 K (d) 3 109 K (c) carbon-12 (d) nitrogen-12
1 22. An electron is moving in an orbit of a hydrogen
12. An alpha nucleus of energy mv 2 bombards a atom from which there can be a maximum of six
2
heavy nuclear target of charge Ze. Then the distance transitions. An electron is moving in an orbit of
of closest approach for the alpha nucleus will be another hydrogen atom from which there can be
proportional to a maximum of three transitions. The ratio of the
1 1 velocities of the electron in these two orbits is
(a) v (b) (c) 4 (d) Ze2
m2 v 1 2 5 3
(a) (b) (c) (d)
13. The radioactivity of a certain material drops to 1/16 2 1 4 4
of the initial value in 2 h. The half life of this radio 27
23. An a-particle of mass 6.4 10 kg and charge
nuclide is 3.2 1019 C is situated in a uniform electric field
(a) 10 min (b) 20 min (c) 30 min (d) 40 min of 1.6 105 V m1. The velocity of the particle at the
14. The de Broglie wavelength of the electron in the end of 2 102 m path when it starts from rest is
ground state of the hydrogen atom is (Radius of the (a) 2 2 105 m s 1 (b) 8 105 m s1
first orbit of hydrogen atom = 0.53 )
(a) 1.67 (b) 3.33 (c) 1.06 (d) 0.53 (c) 16 105 m s1 (d) 4 2 105 m s1
15. At two points P and Q on screen in Youngs double 24. Two thin lenses have a combined power of +9 D.
slit experiment, waves from slits S1 and S2 have a When they are separated by a distance of 20 cm,
l 27
path difference of 0 and respectively. The ratio of their equivalent power becomes + D . Their
4 individual powers (in dioptre) are 5
intensities at P and Q will be (a) 4, 5 (b) 3, 6 (c) 2, 7 (d) 1, 8

Physics for you | march 16 17


25. Two beams of red and violet colours are made to 33. The wavelength of the light used in Youngs double
pass separately through a prism of angle 60. In the slit experiment is l. The intensity at a point on the
minimum deviation position, the angle of refraction l
inside the prism will be screen is I, where the path difference is . If I0
6
(a) greater for red colour denotes the maximum intensity, then the ratio of
(b) equal but not 30 for both the colours I and I0 is
(c) greater for violet colour (a) 0.866 (b) 0.5 (c) 0.707 (d) 0.75
(d) 30 for both the colours. 34. Two media having speeds of light 2 108 m s1 and
26. Of the following transitions in the hydrogen atom, 2.4 108 m s1, are separated by a plane surface.
the one which gives an emission line of the highest What is the angle for a ray going from medium I to
frequency is medium II?
(a) n = 1 to n = 2 (b) n = 2 to n = 1 1 5 1 5
(c) n = 3 to n = 10 (d) n = 10 to n = 3 (a) sin (b) sin
6 12
27. In Millikans oil drop experiment, a charged drop
1 1 1 1
of mass 1.8 1014 kg is stationary between the (c) sin (d) sin
2 2
plates. The distance between the plates is 0.9 cm
and potential difference between the plates is 35. The figure shows
2000 V. The number of electrons in the oil drop is variation of photocurrent
(a) 10 (b) 5 (c) 50 (d) 20 with anode potential
for a photo-sensitive
28. In CE mode, the input characteristics of a transistor
surface for three different
is the variation of
radiations. Let Ia, Ib and Ic
(a) IB against VBE at constant VCE
be the intensities and fa , fb
(b) IC against VCE at constant VBE
and fc be the frequencies
(c) IB against IC (d) IE against IC.
for the curves a, b and c respectively. Then
29. The radioactivity of a sample is I1 at a time t1 and I2
at a time t2. If the half life of the sample is t1/2, then solution of february 2016 crossword
the number of nuclei that have disintegrated in the
time (t2 t1) is proportional to
(a) I1 t2 I2 t1 (b) I1 I2
I1 I 2
(c) (d) (I1 I2) t1/2
t1/ 2
30. A particle of mass M at rest decays into two masses
m1 and m2 with non-zero velocities. The ratio of
l
de Broglie wavelengths of the particles 1 is
l2
m2 m1 m1
(a) (b) (c) (d) 1
m1 m2 m2
31. A fish at a depth of 12 cm in water is viewed by
an observer on the bank of a lake. To what height
the image of the fish is raised? (Refractive index of
water = 4/3) Winners (February 2016)
(a) 9 cm (b) 12 cm (c) 3.8 cm (d) 3 cm Amey Gupta (UP)
rohit Garg (Haryana)
32. The transition from the state n = 4 to n = 1 in a

hydrogen like atom results in ultraviolet radiation. solution senders (January 2016)
Infrared radiation will be obtained in the transition Harsh Verma (UP)
from Mayank Kumar (Bihar)
(a) 2 1 (b) 3 2 (c) 4 2 (d) 5 3 Lovedeep singh (Punjab)

18 Physics for you | march 16


(a) fa = fb and Ia Ib (b) fa = fc and Ia = Ic 4
(c) fa = fb and Ia = Ib (d) fb = fc and Ib = Ic (a) 4 l (b) 8 l (c) l (d) 6 l
3
36. If gE and gM are the acceleration due to gravity on 41. For compound microscope, f0 = 1 cm, fe = 2.5 cm.
the surfaces of the earth and the moon respectively An object is placed at distance 1.2 cm from object
and if Millikans oil drop experiment could be lens. What should be length of microscope for
performed on the two surfaces, one will find the normal adjustment?
electronic charge on the moon (a) 8.5 cm (b) 8.3 cm (c) 6.5 cm (d) 6.3 cm
ratio to be 42. In Youngs double slit interference pattern, the
electronic charge on the earth
g g fringe width
(a) 1 (b) 0 (c) E (d) M (a) can be changed only by changing the wavelength
gM gE
of incident light
37. In common emitter amplifier, the current gain is (b) can be changed only by changing the separation
62. The collector resistance and input resistance are between the two slits
5 kW and 500 W respectively. If the input voltage is (c) can be changed either by changing the
0.01 V, the output voltage is wavelength or by changing the separation
(a) 0.62 V (b) 6.2 V (c) 62 V (d) 620 V between two sources
38. A thin convex lens of crown glass having refractive (d) is a universal constant and hence cannot be
index 1.5 has power 1 D. What will be the power of changed.
similar convex lens but refractive index 1.6? 43. An a-particle and a proton are accelerated from rest
(a) 0.6 D (b) 0.8 D (c) 1.2 D (d) 1.6 D by a potential difference of 100 V. After this, their
39. When a monochromatic point source of light is de Broglie wavelengths are la and lp respectively.
at a distance 0.2 m from a photoelectric cell, the lp
The ratio to the nearest integer, is
saturation current and cut-off voltage are 12.0 la
mA and 0.5 V respectively. If the same source is (a) 3 (b) 4 (c) 6 (d) 5
placed 0.4 m away from the photoelectric cell, then
44. If the binding energy of the electron in a hydrogen
the saturation current and the stopping potential
atom is 13.6 eV, the energy required to remove the
respectively are electron from the first excited state of Li2+ is
(a) 4 mA and 1 V (b) 12 mA and 1 V (a) 30.6 eV (b) 13.6 eV
(c) 3 mA and 1 V (d) 3 mA and 0.5 V (c) 3.4 eV (d) 122.4 eV
40. When a piece of metal is illuminated by a 45. If l is the wavelength of hydrogen atom from the
monochromatic light of wavelength l, then stopping transition n = 3 to n = 1, then what is the wavelength
potential is 3 VS. When same surface is illuminated for doubly ionised lithium ion for same transition?
by light of wavelength 2l, then stopping potential
becomes VS. The value of threshold wavelength for l l
(a) (b) 3 l (c) (d) 9 l
photoelectric emission will be 3 9

ANSWER KEYS
SET 1
1. (d) 2. (a) 3. (d) 4. (b) 5. (d) 6. (b) 7. (c) 8. (c)
9. (b) 10. (d) 11. (a) 12. (b) 13. (a) 14. (c) 15. (c) 16. (a)
17. (d) 18. (d) 19. (b) 20. (a) 21. (d) 22. (d) 23. (b) 24. (b)
25. (c) 26. (b) 27. (b) 28. (c) 29. (c) 30. (b) 31. (c) 32. (d)
33. (d) 34. (d) 35. (a) 36. (d) 37. (d) 38. (c) 39. (d) 40. (a)
41. (c) 42. (a) 43. (a) 44. (b) 45. (b)
SET 2
1. (b) 2. (a) 3. (b) 4. (b) 5. (c) 6. (a) 7. (c) 8. (b)
9. (d) 10. (d) 11. (a) 12. (d) 13. (c) 14. (b) 15. (b) 16. (a)
17. (b) 18. (b) 19. (d) 20. (a) 21. (c) 22. (d) 23. (d) 24. (b)
25. (d) 26. (b) 27. (b) 28. (a) 29. (d) 30. (d) 31. (d) 32. (d)
33. (d) 34. (a) 35. (a) 36. (a) 37. (b) 38. (c) 39. (d) 40. (a)
41. (b) 42. (c) 43. (a) 44. (a) 45. (c) nn

Physics for you | march 16 19


Have you ever given it a thought that- 2. F^ : The component of the applied force which is
Why are door knobs always attached towards the perpendicular to the position vector.
extreme end, far away from the hinged end? This component clearly can change the orientation,
hence we should try to maximise its value.
Therefore keep in mind, whenever we talk of rotation

capability of F , we think of F ^ .

It is also observed that if we fix F , and only change
the point of application, increasing r increases rotation
capability and decreasing r decreases the rotation
capability.

Why are the handle bars of wrenches made large? Hence the rotation capability of F , i.e., the torque of

F with respect to origin is
t = rF^ = rF sinq = (r sinq)F = r^ F
where r ^ can be seen as the
component of position vector
which is perpendicular to the
Now that you have started thinking, the obvious answer applied force.
that you come across is that it becomes easier to rotate
The rotation will also have a
them. But why it is so?
direction, either clockwise or
The answer is TORQUE!
anticlockwise with respect to
Torque of an applied force represents the rotational
an observer.
capability of the applied force to rotate the line joining
For example in the above example the torque is
the point of application of the applied force and the
anticlockwise.
axis of rotation (AOR).
In vector representation, we can
Let us see for example, a
find it by using right hand curl
force F being applied on a rule where we curl the fingers
point object whose position of the right hand in the sense

vector is r as given here. of rotation keeping the thumb

Is this F capable of changing straight, and the direction in

the orientation of r , i.e., is which thumb points gives us the
it capable of changing f? To direction of the axis of rotation (AOR), somewhat as
understand this we break given here.
Let the plane of paper be the xy plane. Hence we
two components of the applied force F
use following conventions to represent clockwise/
1. F|| : The component of the applied force which is
anticlockwise rotation.
parallel to the position vector.
1. Clockwise:
This component clearly cannot change the
orientation, it changes the distance r. or or (k )
Contributed By: Bishwajit Barnwal, Aakash Institute, Kolkata

20 physics for you | MarcH 16


2. Anticlockwise:
Correlating it with linear momentum p = mv , it is
or or (k ) clear that MOI has the same role to play in rotational
In vector form, torque of the applied force is mechanics which is played by mass in translational

t=rF mechanics.
Remember that torque of a force is axis/point of In this article, I am assuming, that you know the
standard MOI results of different types of objects.
observation dependent, since r is the position vector of
3. Translation + Rotation :
the point with respect to origin which was our point of
\ With respect to origin
observation. Hence changing it means changing t .
So, when can the torque of a force be zero? L = r COM mv COM + ICOM w
Two cases
L = r COM mv COM + ICOM w .
1. Applied force passes through the point of

observation/AOR. r F = 0 Relation between torque and angular momentum

2. Applied force is parallel to AOR. L=r p
The door is hinged at one of its sides
which behaves as AOR and the weight dL d p dr
\ =r + p = r F + (v p)
mg tends to turn its AOR itself which dt dt dt

is fixed. Hence mg will not create any
torque here.
{
= r F v is parallel to p

}
dL
Angular momentum (L) \t= {Newton's 2nd law in rotational mechanics.}
It is a measure of the amount of rotational motion of dt
an object with respect to an observation point in the For an object rotating about an axis,
same way as linear momentum is seen as the amount
L AOR = I AOR w
of translational motion in the object.
It is measured in 3 different ways depending upon the dL AOR dw
type of motion. = I AOR t AOR = I AOR a
dt dt
1. Pure translation:
d w
L = r p = r (mv ) = m(r v ) where a = = angular acceleration.
dt
where r is the position This relation is similar to F = ma of translational
vector of the COM mechanics.
with respect to point Hence the equation says that the torque of all the forces
of observation. acting on the object with respect to AOR is equal to
\ With respect to the product of MOI about the chosen axis multiplied
origin, with angular acceleration.
L = mvr sinq Pure Rolling
= m(v sinq)r or mv(r sinq) It is a special case of translation and rotation of an
= mv^r or mvr^ object where the point of contact of the object does
2. Pure rotation : not slip over the surface on which it is kept which is
The velocity of an elemental mass dm, possible only if the velocity of the object at the point
v = wr. of contact is same as the surface. Let us see an example
\ With respect to AOR, to understand better.
dLAOR = (dm)(wr)(r) The sphere of radius R is
\ dLAOR = (dmr2)w rolling on a plank which is
\ LAOR = dLAOR =

( dmr 2 ) w also moving. Let us find a
condition for pure rolling.
\ L AOR = I AOR w We have chosen two points
A and B, one on the rolling
where I AOR = dmr 2 represents the moment of inertia object and the other on the plank. Both these points are
(MOI) about the chosen axis of rotation. in contact.

physics for you | MarcH 16 21


\ For pure rolling, Now before we start solving questions, remember that
vA = vB for torque calculations, torque is axis specific. Hence you
can choose any arbitrary point for applying t = Ia, but
we prefer those points through which maximum number
of unknown forces pass through. The advantage is that
the torque of such forces will be zero. But be careful
in the selection since, it should not be an accelerated
point else we will have to consider a pseudo force on the
\ v-wR = vp object which would pass through the COM and might
This is the required condition! have a torque of its own. Whenever in confusion about
What if the sphere was rolling on a fixed surface? the selection of the point; prefer COM, since even if
In such case, vp = 0 it is accelerated, pseudo force will pass through it and
\ v = wR. hence would not have any torque.
\ v = wR is not the condition for pure rolling in all Now, Let us solve some questions.
cases, it is only when the surface is fixed. Q1: A string is wrapped around a disc
Let us consider an object and one end of string tied to a ceiling
rolling on ground with and released. Find the acceleration of
instantaneous values of linear the COM.
velocity of COM vCOM, angular Soln.: For translational mechanics,
velocity w, linear acceleration mg T = maCOM ...(i)
of COM a COM and angular For rotational mechanics, I prefer the
acceleration a as shown. point of contact P, which is on the
Note: To differentiate between velocity and acceleration, straight string, since, the torque of
I have shown velocity with straight tail and acceleration tension would be zero, hence we get
with zig-zag tail. a in one equation directly
Let us find out the velocity and acceleration of any \ tp = Ipa
arbitrary point on the sphere. 3 2 2
To find the acceleration, mgR = mR a aR = g ...(ii)
2 3
we first find the Now, aCOM and a are related, and
acceleration of each the relation between them is the
point with respect to constraint relation.
the COM and then we The acceleration of point P is zero.
add the acceleration of Hence,
COM vectorially to each \ ap = 0 aCOM aR = 0
of these points.
2
Remember that with aCOM = aR = g
respect to COM each 3
Q2: A solid sphere is projected by
point on sphere has
tangential as well as giving it a translational velocity v0
radial acceleration. on a rough horizontal surface with
friction coefficient m as shown.
Find the velocity of COM after pure rolling starts.
Soln.: Since slipping star ts,
kinetic friction would act in
backward direction due to which
COM decelerates and sphere
attains angular acceleration in
anticlockwise direction as shown.
Note here that, the point of contact is tangentially fk = maCOM
unaccelerated but is radially accelerated. mmg = maCOM aCOM = mg ...(i)

22 physics for you | MarcH 16


With respect to COM, \ PEloss = KEgain
tCOM = ICOM a l 1 ml 2 2 . .
mg = w { . it is a case of pure rotation, so
2 2 5mg 2 2 3 only rotational KE about AOR}
(mmg )R = mR a aR = ...(ii)
5 2 wl = 3 gl = velocity of free end.
\ Linear velocity (v) and angular velocity (w),
t seconds later are, Q4: On releasing the rod of mass m and length l from
v = u + aCOM t = v0 mgt ...(iii) the position shown, it is found that rod rotated about
w = w0+ at the extreme edge of the table and started slipping after
5mg turning through an angle of 37. Find the friction
= t (... w0 = 0) ...(iv) coefficient between table and rod.
2R
When pure rolling starts,
v = wR
5 7 2 v0
v0 mgt = mgt v0 = mgt t =
2 2 7 mg
Soln.: Again till slipping does not start, friction cannot
2 5
\ v = v0 mgt = v0 v0 = v0 perform any net work.
7 7
Alternatively, let me show you a smart way to solve
this question.

3l 1 ml 2 l
2
mg = + m w2
About the point P on ground, arbitrarily chosen, 20 2 12 4
there is no external torque, hence angular momentum 3 gl 7 2 2 72
2
remains conserved. = w l w l= g ...(i)
10 48 35
LP initially = LP finally For rotational motion, about the point of contact
mv0r = mvr + ICOM w t = Ia
l ml 2 l
2 2 5 2
= mvr + mR2 w = mvr + mvr v = v0 (mg cos 37) = +m a
5 5 7 4 12 4
Q3: The rod is hinged at one 2
l 7ml 48 g
end and released by disturbing mg = a al = ...(ii)
5 48 35
it from its unstable equilibrium Along the normal,
position. Find the speed of the al
free end when the rod become mg cos 37 N = maCOM = m
4
horizontal.
4 m 48 g 12mg
Soln.: The only forces acting on the rod are normal force mg N = =
exerted at the hinge and weight. The point of application 5 4 35 35
of normal force is always at instantaneous rest hence it 4 12 16
N = mg = mg ...(iii)
cannot perform any net non zero work. Hence only gravity 5 35 35
is performing work. Along the length of rod,
l
mg sin 37 f k = mw2
4
3 16mg m 72 g 16 3 18 3
mg m = m= =
5 35 4 35 35 5 35 35
3
m=
16
nn
physics for you | MarcH 16 23
Exam Dates
OfflinE : 3rd April
OnlinE : 9th & 10th April

2d
1. A spaceship is launched into a circular orbit close (a) h + (b) 2h + 2d
m
to earth's surface. The additional velocity that d
(c) h + d (d) h +
should be imparted to the spaceship in the orbit to m
overcome the gravitational pull is 5. Two particles A and B having charges 8 106 C
(Radius of earth = 6400 km and g = 9.8 m s2) and 2 106 C respectively, are held fixed with a
(a) 11.2 km s1 (b) 8 km s1 separation 20 cm. Where should a third charged
(c) 3.2 km s1 (d) 1.5 km s1 particle be placed so that it does not experience a
net electric force?
2. A hole is drilled in a copper sheet. The diameter of
(a) 0.2 m from B (b) 0.5 m from A
the hole is 4.24 cm at 27.0C. What is the change in
(c) 0.6 m from A (d) 0.1 m from B
the diameter of the hole when the sheet is heated to
227C? Coefficient of linear expansion of copper is 6. Two blocks M1 and M2 having equal mass are to
1.70 105C1. move on a horizontal frictionless surface. M2 is
(a) 1.44 102 cm (b) 2.44 103 cm attached to a massless spring as shown in figure.
(c) 1.44 10 mm (d) 2.44 103 mm
2 Initially M2 is at rest and M1 is moving toward M2
with speed v and collides head-on with M2.
3. The number density of free electrons in a copper
conductor estimated is 8.5 1028 m3. How long
does an electron take to drift from one end of a
wire 3.0 m long to its other end? The area of cross-
section of the wire is 2.0 106 m2 and it is carrying
(a) While spring is fully compressed, all the kinetic
a current of 3.0 A.
energy of M1 is stored as potential energy of
(a) 6 h 23 min (b) 7 h 33 min
spring.
(c) 7 h 43 min (d) 6 h 53 min
(b) While spring is fully compressed the system
4. A point luminous object (O) is at a distance h from momentum is not conserved, though final
front face of a glass slab of width d and of refractive momentum is equal to initial momentum.
index m. On the back face of slab is a reflecting (c) If spring is massless, the final state of the M2 is
plane mirror. An observer sees the image of object state of rest.
in mirror [figure]. Distance of image from front face (d) If the surface on which blocks are moving has
as seen by observer will be friction, then collision cannot be elastic.

Observer 7. A block of mass m = 2 kg is resting on a rough


O inclined plane of inclination 30 as shown in figure.
h
The coefficient of friction between the block and
the plane m = 0.5. What minimum force F should
d
be applied perpendicular to the plane on the block,
so that block does not slip on the plane? ((gg = 10 m s2)

24 Physics For you | March 16


(a) (b)

(a) 2.68 N (b) Zero


(c) 4.34 N (d) 6.24 N
8. The density of a solid ball is to be determined in an
experiment. The diameter of the ball is measured (c) (d)
with a screw gauge, whose pitch is 0.5 mm and
there are 50 divisions on the circular scale. The
reading on the main scale is 2.5 mm and that on the 12. In a metre bridge experiment null point is obtained
circular scale is 20 divisions. If the measured mass at 20 cm from one end of the wire when resistance
of the ball has a relative error of 2%, the relative X is balanced against another resistance Y. If
percentage error in the density is
X < Y, then where will be new position of the null
(a) 0.9% (b) 2.4%
point from the same end, if one decides to balance
(c) 3.1% (d) 4.2%
a resistance of 4X against Y?
9. A carpet of mass M, made of an extensible material
(a) 50 cm (b) 80 cm (c) 40 cm (d) 70 cm
is rolled along its length in the form of a cylinder of
radius R and kept on a rough floor. If the carpet is 13. When a metal surface is illuminated with light of
unrolled, without sliding to a radius R/2, the decrease wavelength l, the stopping potential is V0. When
in potential energy is the same surface is illuminated with light of
1 7 V0
(a) MgR (b) MgR wavelength 2l, the stopping potential is . If the
2 8 4
5 3 velocity of light in air is c, the threshold frequency
(c) MgR (d) MgR of photoelectric emission is
8 4
10. A liquid of density r0 is filled in a wide tank to a c c 2c 4c
(a) (b) (c) (d)
height h. A solid rod of length L, cross-section A 6l 3l 3l 3l
and density r is suspended freely in the tank. The 14. Two identical capacitors 1 and 2 are connected in
lower end of the rod touches the base of the tank and series to a battery as shown in figure. Capacitor 2
h = L/n (where n > 1). Then the angle of inclination contains a dielectric slab of dielectric constant K
q of the rod with the horizontal in equilibrium as shown. Q1 and Q2 are the charges stored in the
position is capacitors. Now the dielectric slab is removed and
r r the corresponding charges are Q1 and Q2. Then
(a) sin 1 0 (b) sin 1 n 0
r r Q 1 K + 1 K
(a) =
1 r0 1 r Q1 K 1 2
(c) sin 1 (d) sin 1
n r n r0 Q 2 K + 1
(b) =
11. A cyclic process ABCA shown in V-T diagram, is Q2 2 V

performed with a constant mass of an ideal gas. Q 2 K + 1 Q 1 K


Which of the following graphs in figure represents (c) = (d) =
Q2 2K Q1 2
the corresponding process on a P-V diagram?
15. Two pendulums differ in lengths by 22 cm.
They oscillate at the same place so that one of
them makes 30 oscillations and the other makes
36 oscillations during the same time. The lengths
(in cm) of the pendulums are
(a) 72 and 50 (b) 60 and 38
(c) 50 and 28 (d) 80 and 58

Physics For you | March 16 25


16. An inductance coil is connected to an ac source 2p 3
through a 60 W resistance in series. The source (c) (d)
3 2p
voltage, voltage across the coil and voltage across
22. A body of mass m thrown horizontally with velocity
the resistance are found to be 33 V, 27 V and
v, from the top of tower of height h touches the
12 V respectively. Therefore, the resistance of the coil is
level ground at distance of 250 m from the foot of
(a) 30 W (b) 45 W (c) 105 W (d) 75 W
the tower. A body of mass 2 m thrown horizontally
17. A galvanometer of 50 W resistance has 25 divisions. v
A current of 4 10 4 A gives a deflection of one with velocity , from the top of tower of height 4 h
2
division. To convert this galvanometer into a
will touch the level ground at a distance x from the
voltmeter having a range of 25 V, it should be
foot of tower. The value of x is
connected with a resistance of
(a) 250 m (b) 500 m
(a) 2500 W as a shunt (b) 2450 W as a shunt
(c) 2550 W in series (d) 2450 W in series (c) 125 m (d) 250 2 m
18. A boy of mass 30 kg starts running from rest along a 23. The Poission's ratio of a material is 0.4. If a force is
circular path of radius 6 m with constant tangential applied to a wire of this material, there is a decrease
acceleration of magnitude 2 m s2. After 2 s from of cross-sectional area by 2%. The percentage
start he feels that his shoes started slipping on increase in its length is
ground. The friction between his shoes and ground is (a) 3% (b) 2.5%
(Take g = 10 m s2) (c) 1% (d) 0.5%
1 1 1 1
(a) (b) (c) 4 (d) 5 24. When an AC source of emf e = e0 sin(100 t) is
2 3
19. Two short bar magnets of magnetic moment M each connected across a circuit, the phase difference
are arranged at the opposite corners of a square of between emf (e) and current (i) in the circuit is
side d such that their centres coincide with the p
observed to be , as shown in figure. If the circuit
corners and their axes are parallel. If the like poles 4
are in the same direction, the magnitude of the consist possibly only of RC or RL in series, find the
magnetic induction at any of the other corners of relationship between the two elements.
the square is
m0 M m0 2M
(a) 4 p 3 (b) 4 p 3
d d
m0 M m0 M 3
(c) (d)
4 p 2d 3 4 p 2d 3
20. A long glass tube is held vertically in water. A (a) R = 1 kW, C = 5 mF (b) R = 1 kW, C = 10 mF
tuning fork is struck and held over the tube. Strong (c) R = 1 kW, C = 1 H (d) R = 1 kW, L = 10 H
resonances are observed at two successive lengths
25. A light ray from air is incident as shown in figure
0.50 m and 0.84 m above the surface of water. If
velocity of sound is 340 m s1, then the frequency at one end of the glass fibre making an incidence
of the tuning fork is angle of 60 on the lateral surface, so that it just
(a) 128 Hz (b) 256 Hz undergoes a total internal reflection. How much
(c) 384 Hz (d) 500 Hz time (in ms) would it take to traverse the straight fibre
of length 1 km?
21. A particle executes SHM with an amplitude of
2 cm. When the particle is at 1 cm from the mean
position, the magnitude of its velocity is equal to
that of its acceleration. Then its time period in
seconds is
1
(a) (b) 2 p 3 (a) 3.85 (b) 4.25 (c) 2.90 (d) 7.30
2p 3

26 Physics For you | March 16


26. If a zener diode (VZ = 5 V and IZ = 10 mA) is For escaping from closed to the surface of earth,
connected in series with a resistance and 20 V is GMm 1 2
= mve
applied across the combination, then the maximum R 2
resistance one can use without spoiling zener action is 2GM
(a) 20 kW (b) 15 kW ve = = 2 gR
R
(c) 10 kW (d) 1.5 kW
ve = 2 vo = 1.41 8 = 11.2 km s 1
27. An electromagnetic wave travels along z-axis. \ The additional velocity to be imparted to the
Which of the following pairs of space and time orbiting satellite for escaping is 11.2 8 = 3.2 km s1
varying fields would generate such a wave?
2. (a) : Given, diameter of the hole, d1 = 4.24 cm
(a) Ex, By (b) Ez, Bx
Initial temperature, T1 = 27 + 273 = 300 K
(c) Ey, Bz (d) Ey, Bx Final temperature, T2 = 227 + 273 = 500 K
28. In the circuit as shown in figure, the current in Coefficient of linear expansion, a = 1.70 105C1
ammeter is Coefficient of superficial expansion, b = 2a
3V 2 = 3.40 105C1
pd12
10 Area of hole at 27C, A1 = pr2 =
A 4
p
1 = (4.24)2 = 4.494 p cm2
4
6V
Area of hole at 227C, A2 = A1(1 + b DT)
42
A (b)
27 15 32
A = 4.494p[1 + 3.40 105 (227 27)]
(a) A (c) A (d)
32 32 32 15 = 4.494p[1 + 3.40 105 200]
29. In Youngs double slit experiment, one of the slits is = 4.494p 1.0068
wider than the other, so that the amplitude of the = 4.525p cm2
light from one slit is double than that from the other If diameter of hole becomes d2 at 227C, then
slit. If Im be the maximum intensity, the resultant pd22
intensity when they interfere at a phase difference f A2 =
4
is given by pd22
Im Im 4.525p =
2 f 2 f 4
(a) 1 + 2 cos (b) 1 + 4 cos
3 2 5 2 or 2
d2 = 4.525 4 or d2 = 4.2544 cm
Im 2 f I m 2 f
(c) 1 + 8 cos (d) 8 + cos \ Change in diameter, Dd = d2 d1
9 2 9 2 = 4.2544 4.24 = 0.0144 cm = 1.44 102 cm
30. What is the minimum thickness of a thin film 3. (b) : Given,
required for constructive interference in the Number density of electrons, n = 8.5 1028 m3
reflected light from it ? Given, the refractive index Length of wire, l = 3 m
of the film = 1.5, wavelength of the light incident on Area of cross-section of wire, A = 2 106 m2
the film = 600 nm. Current I = 3 A
(a) 100 nm (b) 300 nm (c) 50 nm (d) 200 nm Charge on electron, e = 1.6 1019 C
solutions Time taken by electron to drift from one end to
1. (c) : For spaceship orbiting close to earth's surface another of the wire,
Length of the wire l
mvo2 GMm t= = ...(i)
= Drift velocity vd
R R2
Using the relation,
GM
i.e., vo = = gR I = neAvd
R I
or vd = ...(ii)
\ vo = (9.8 6.4 106 ) 8 km s 1 neA

Physics For you | March 16 27


Putting the value from eq. (ii) in eq. (i),
\ F C = F CA + F CB
l neA 28 19 6
3 8.5 10
1.6 10 2 10
t= = 1 (8 106 )Q (2 106 )Q
I 3 = i
4 pe0 (0.2 + x )2 x2
or t = 2.72 104 s = 7 h 33 min

Thus, the time taken by an electron to drift from But | F C | = 0
one end to another end is 7 h 33 min.
1 (8 106 )Q (2 106 )Q
4. (a) : As shown in figure Then =0
4 pe0 (0.2 + x )2 x2
glass slab will form the
which gives, x = 0.2 m
image of bottom i.e.,
mirror MM at a depth 6. (d) : While spring is fully compressed, the entire
d kinetic energy of M1 is not stored as potential energy
m from its front face. of spring as M2 may move. If spring is massless, also
M1 = M2, velocities of M1 and M2 are interchanged
So the distance of object
on collision. M1 comes to rest, instead of M2. If
O from virtual mirror surface on which blocks are moving has friction,
d loss of energy is involved. Collision cannot be elastic.
mm will be h + . Choice (d) is correct.
m
Now as a plane mirror forms image behind the 7. (a) : mg sin q = 2 10 sin 30 = 10 N
mirror at the same distance as the object is in front and f = m R = m mg cos q
of it, the distance of image I from mm will be = 0.5 2 10 cos 30
d = 10 0.866 = 8.66 N
h + m and as the distance of virtual mirror from As mg sin q > f, the block
d tends to slip down the plane.
the front face of slab is m , the distance of image On applying F perpendicular to plane,
I from front face as seen by observer will be R = F + mg cos 30
d d 2d To avoid slipping,
= h + + = h + mg sin 30 = m R = m(F + mg cos 30)
m m m
\ 2 10 1 / 2 3
F= 2 10
5. (a) : The net electric force on C should be equal to 0. 5 2
zero, the force due to A and B must be opposite in F = 20 17.32
direction. Hence, the particle should be placed on = 2.68 N
the line AB. As, A and B have charges of opposite 8. (c) : Least count of screw gauge
nature, also A has larger magnitude of charge than B. pitch
=
Hence, C should be placed closed to B than A. From no. of division on circular scale
figure BC = x (say) and charge on C is Q.
0.5 mm
1 (8 106 )Q = = 0.01 mm.
Then, F CA = i 50
4 pe0 (0.2 + x )2 Diameter of ball, D = MSR + CSR LC
1 (2 106 )Q = 2.5 mm + 20 0.01 mm = 2.7 mm
and F CB = i
4 pe0 x2 mass M
As density, r = =
volume 4 p D 3

3 2
Relative error in the density,
Dr DM 3DD
= +
r M D
Relative percentage error in the density is

28 Physics For you | March 16


Dr DM 3DD \ P is constant (A B is a straight line || to volume axis).
100 = + 100 From B to C, volume V is constant (B C is a straight
r M D
line || to pressure-axis).
DM 3DD 0.01
= 100 + 100 = 2 + 3 100 From C to A, temperature T is constant.
M D 2. 7 \ PV = constant (Boyle's law)
= 2% + 1.11% = 3.1% 1
9. (b) : The centre of mass of the whole carpet is originally So, CA is a curve such that P .
V
at a height R above the floor. When the carpet unrolls Hence, correct representation is in figure (a).
itself and has a radius R/2, the centre of mass is at a
X 20 20 1
height R/2. The mass left over unrolled is 12. (a) : In the first case, = = =
Y (100 20) 80 4
M p(R/2)2 M or Y = 4X ... (i)
=
2 4 4X l
pR In the second case, =
\ The decrease in potential energy Y 100 l
M R 7 4X l
= MgR g = MgR or = (Using (i))
4 2 8 4 X 100 l
10. (c) : Refer to figure, let l be the length of rod or l = 50 cm
immersed in liquid. q be the angle of inclination of 13. (b) : According to Einsteins photoelectric equation
rod with horizontal in equilibrium position. hu = Kmax + f0
hc
= eVs + f0
l
where, l = wavelength of incident light
f0 = work function
Vs = stopping potential
According to given problem
hc
The weight of rod = mg = ALrg acting vertically = eV0 + f0 ...(i)
l
downwards at the centre of gravity C of the rod. hc eV0
The upward thrust on rod, FB = Alr0g acting = + f0 ...(ii)
2l 4
vertically upwards at the centre of buoyancy D;
Subtract (ii) from (i), we get
which is the mid point of length of rod inside the
hc 1 1
liquid. 1 = eV0 1
l 2 4
As the rod is in equilibrium position, then net
hc 3 2 hc
torque on the rod about point A is zero, i.e., = eV0 or eV0 =
L l 2l 4 3 l
( AL r g ) cos q ( Al r0 g ) cos q = 0 Substituting the value of eV0 in eq. (i), we get
2 2
hc 2 hc hc
L2 r0 L r0 = + f0 or f0 =
or = or = l 3 l 3l
l2 r l r \ Threshold frequency
h L / n 1 L 1 r0 f hc c
Now, sin q = = = = u0 = 0 = =
l l nl n r h 3lh 3l
1 r0 14. (c) : Let C be the capacitance of capacitor without
or q = sin 1 slab.
n r
Before the slab is removed
V C1 = C and C2 = KC
11. (a) : From A to B, V T or = constant
T CC (C )(KC ) K
PV \ Cnet = 1 2 = =
As = R = constant C
T C1 + C2 C + KC K + 1

Physics For you | March 16 29


KCV This resistance of 2450 W should be connected in
\ Q1 = Q2 = series to convert the galvanometer into a voltmeter.
K +1
After the slab is removed, C1 = C and C2 = C 18. (b) : After 2 s, speed of boy will be
(C )(C ) C \ Q = Q = CV v = 2 2 = 4 m s1
\ Cnet = = 1 2 At this moment, centripetal force on the boy is
C +C 2 2
mv 2 30 16
Q 1 Q 2 K +1 Fc = = N = 80 N
R 6
Hence, Q = Q = 2K
1 2 Tangential force on the boy is
15. (a) : Time period of a pendulum is Ft = ma = 30 2 N = 60 N
l T l Total force acting on boy is
T = 2p \ 1= 1
g T2 l2 F = Fc2 + Ft2 = (80)2 + (60)2 = 100 N
T N At the time of slipping, F = mmg
Also, 1 = 2 where N1 = 30 and N2 = 36
T2 N1 1
or 100 = m 30 10 or m =
l1 N 2 36 3
\ = = ... (i) 19. (a) :
l2 N1 30
Also, l1 l2 = 22 cm ... (ii)
Solving the eqs. (i) and (ii), we get
l1 = 72 cm and l2 = 50 cm
16. (b) : L
r R = 60

27 V 12 V

Magnetic induction at point E due to magnet at F


33 V
m 2M
Let r be resistance of the coil. (axial point) is B1= 0
12 V 4p d3
Current in the circuit, I = = 0. 2 A It acts along EF.
60 W Magnetic induction at point E due to magnet at D
According to voltage formula m M
272 = VL2 + Vr2 (equatorial point) is B2 = 0
... (i) 4p d3
33 2
= VL2
+ (VR + Vr ) 2 ... (ii) It acts along FE.
Resultant magnetic induction (magnitude) at point E is
Subtract (i) from (ii), we get m M
332 272 = VR2 + Vr2 + 2VRVr Vr2 B = B1 B2 = 0
4 pd 3
332 272 = 122 + 2 12 Vr 20. (d) : From v = 2u(l2 l1)
332 272 122 v 340 340
Vr = =9V u= = =
(2 12) 2(l2 l1 ) 2(0.84 0.50) 2 0.34 = 500 Hz
Vr 9V
Vr = Ir r = = = 45 W 21. (c) : Here, A = 2 cm
I 0.2 A Magnitude of velocity from mean position
17. (d) : G = 50 W 2 2
Ig = Current for full scale deflection = w A x
= Current per division total no. of divisions and acceleration = w2x.
= 4 104 25 = 102 A
Now, w2 x = w A2 x 2 or, w2 1 = w 4 1
Given V = 25 V
Hence, required resistance, or, w = 3.
V 25 2p 2p 2p
R= G = 50 = 2500 50 = 2450 W As w= or T = = s.
Ig 102 T w 3
Contd. on page no. 71

30 Physics For you | March 16


ElEctronic dEvicEs types of energy bands
Energy bands are of two types :
classification of solids Valence band
Solids can be classified on the basis of conductivity and Conduction band
energy bands. For semiconductors and insulators, the energy bands
classification on the basis of conductivity which consist of closely spaced completely filled energy
On the basis of conductivity, solids can be classified in states at 0 K are called valence bands. The bands with
three categories. higher energies are called conduction bands.
Metals : Solids having very high electric conductivity
Forbidden energy gap (Eg) is the energy gap between
are known as metals. Their electric conductivity lies the top of the highest valence band and bottom of the
between 102 S m1 and 108 S m1. lowest conduction band.
Insulators : Solids having very low electric conduc-
On the basis of band theory, solids are classified as follows :
Metals : In metals or good conductors, conduction
tivity are known as insulators. Their conductivity
lies between 1011 S m1 and 1019 S m1. band is either partially filled (figure (a)) or overlaps
Semiconductors : Solids whose conductivity is
(figure (b)) the valence band. There is no forbidden
intermediate to that of metals and insulators are energy gap.
Semiconductors : In semiconductors, the two
known as semiconductors. Their conductivity lies
between 105 S m1 and 106 S m1. energy bands are distinctly separate without any
overlapping.
classification on the basis of energy bands Insulators : The conduction band is empty and
Each electron in an atom has definite energy value. valence band is completely filled.
These definite energy values are called energy levels.
In solids, atoms are closely packed. A solid crystal
contains about 1023 atoms/cm3. So each atom is in the
electrostatic field of neighbouring atoms. So in solids
where atoms are closely spaced, the atomic energy levels
of the electrons broaden and give rise to energy bands.

Physics for you | march 16 31


p -type semiconductor
When a trivalent impurity like B, Al, Ga or In is added
to a pure semiconductor, semiconductor becomes
deficient in electrons, i.e., number of holes become
more than number of electrons. Such a semiconductor
is called p-type semiconductor.
In the p-type semiconductors, holes are majority charge
carriers, whereas electrons are minority charge carriers.
In p-type semiconductor, the fermi level shifts towards
the valence band. The trivalent impurity atoms are
called acceptor atoms.

intrinsic semiconductor
A semiconductor in pure form is called intrinsic
semiconductor. Germanium (Ge) and silicon (Si) are
important examples of intrinsic semiconductors. They
are tetravalent elements. They have covalent bonding.
A pure semiconductor has negative temperature
co-efficient of resistance. At ordinary temperature some n -type semiconductor
electrons absorb energy from lattices and move to the When a pentavalent impurity, such as P, As, Sb or Bi is
conduction band. This happens due to breaking of added to a pure semiconductor, the number of electrons
covalent bond because of the effect of thermal energy. become more than the holes in the semiconductor and
Electrons moving to the conduction band leave behind such a semiconductor is called n-type semiconductor.
the vacancy of electron, called holes, in the valence It has electrons as majority carriers and holes as minority
band. Hole has positive charge equal to that of electron. carriers. In n-type semiconductor, the fermi level shifts
Mobility of hole is smaller than that of electron. towards the conduction band. The pentavalent impurity
The number density (ne) of electrons in conduction atoms are called donor atoms.
band is equal to the number density (nh) of holes in the
valence band ne = nh = ni, where ni is called the intrinsic
carrier concentration.
When electric field is applied across an intrinsic
semiconductor, electrons and holes move in opposite
directions so that conventional current,
I = Ie + Ih ,
where Ie = free electron current and Ih = hole current.
The deliberate addition of a desirable impurity to
intrinsic semiconductor in controlled quantities to Mass Action law
promote conductivity is called doping. In semiconductors due to thermal effect, generation
of free electron and hole takes place. Apart from the
Extrinsic semiconductors process of generation, a process of recombination also
These are obtained by doping the pure semiconductor occurs simultaneously, in which free electron further
with small amount of certain impurities of either recombine with hole.
trivalent or pentavalent atoms. At equilibrium, rate of generation of charge carriers is
Extrinsic semiconductors are of two types: equal to rate of recombination of charge carriers. The
p-type semiconductor recombination occurs due to electron colliding with a
n-type semiconductor hole, larger value of ne or nh, higher is the probability of

32 Physics for you | march 16


their recombination. Hence for a given semiconductor, SELF CHECK
rate of recombination ne nh
so rate of recombination = Rne nh, 1. The current voltage relation of diode is given by
where R = recombination coefficient I = (e1000 V/T 1) mA, where the applied voltage V is
For intrinsic semiconductor, ne = nh = ni in volts and the temperature T is in degree Kelvin. If
a student makes an error measuring 0.01 V while
so rate of recombination = Rni2
measuring the current of 5 mA at 300 K, what will
Rne nh = Rni2 ni2 = ne nh be the error in the value of current in mA?
Under thermal equilibrium, the product of the (a) 0.05 mA (b) 0.2 mA
concentration ne of free electrons and the concentration (c) 0.02 mA (d) 0.5 mA
nh of holes is a constant. Independent of the amount of (JEE Main 2014)
doping by acceptor and donor impurities. depletion region
Mass action law , ne nh = ni2 In the vicinity of junction, the region containing the
uncompensated acceptor and donor ions is known as
conductivity of semiconductor depletion region. There is a depletion of mobile charges
The conductivity of semiconductor is given by (holes and free electrons) in this region. Since this region
s = e(neme + nhmh) has immobile ions which are electrically charged it is
where me and mh are the electron and hole mobilities also known as the space charge region. The electric field
and e is the electronic charge. between the acceptor and the donor ions is known as a
barrier. For a silicon p-n junction, the barrier potential
The conductivity of an intrinsic semiconductor is
is about 0.7 V, whereas for a germanium p-n junction it
si = nie(me + mh) is approximately 0.3 V.
The conductivity of n-type semiconductor is
sn = eNdme KEY POINT
The conductivity of p-type semiconductor is The physical distance from one side of the barrier
sp = eNamh to the other is known as the width of the barrier.
The difference of potential from one side of the
p-n Junction barrier to the other side is known as the height of
When donor impurities are introduced into one side the barrier.
and acceptors into the other side of a single crystal of The width of the depletion layer and magnitude of
an intrinsic semiconductor, a p-n junction is formed. potential barrier depends upon the nature of the
It is also known as junction diode. It is symbolically material of semiconductor and the concentration
represented by of impurity atoms.
The thickness of the depletion region is of the
order of one tenth of a micrometre.
forward Biasing of a p -n Junction
The most important characteristic of a p-n junction is its When the positive terminal of external battery
ability to conduct current in one direction only. In the is connected to p-side and negative to n-side of
other (reverse) direction it offers very high resistance. p-n junction, then the p-n junction is said to be forward
The current in the junction diode is given by biased. In forward biasing, the width of the depletion
I = I0 (eeV/kT 1) region decreases and barrier height reduces. The
where k = Boltzmann constant, I0 = reverse saturation resistance of the p-n junction becomes low in forward
current. biasing.
In forward biasing, V is positive and low, then forward reverse Biasing of a p -n Junction
current, When the positive terminal of the external battery
If = I0 (eeV/kT 1) is connected to n-side and the negative terminal to
In reverse biasing, V is negative and high, eeV/kT < < 1, p-side of a p-n junction, then the p-n junction is said
then reverse current, to be reverse biased. In reverse biasing, the width of the
Ir = I0 depletion region increases and barrier height increases.

Physics for you | march 16 33


The resistance of the p-n junction becomes high in it) when it is forward biased, and as a perfect insulator
reverse biasing. (with no current flow through it) when it is reverse biased.
Breakdown voltage The I-V characteristics of an ideal diode is shown in
A very small current flows through p-n junction, when figure below.
it is reverse biased. The flow of the current is due to
the movement of minority charge carriers. The reverse
current is almost independent of the applied voltage.
However, if the reverse bias voltage is continuously
increased, for a certain reverse voltage, the current An ideal diode acts like an automatic switch.
through the p-n junction will increase abruptly. This
In forward bias, it acts as a closed switch whereas in
reverse bias voltage is thus known as breakdown
reverse bias it acts as an open switch as shown in the
voltage. There can be two different causes for the break
figure below.
down. One is known as Zener breakdown and the other
is known as avalanche breakdown.
I-V characteristics of a p -n Junction
The I-V characteristics of a p-n junction do not obey
Ohms law. The I-V characteristics of a p-n junction are
SELF CHECK
as shown in the figure.
2. The forward biased diode connection is
(a) (b)
(c) (d)
(JEE Main 2014)
3. A 2 V battery is connected
across AB as shown in the
figure. The value of the
current supplied by the
battery when in one case
batterys positive terminal is
connected to A and in other
case when positive terminal
Knee voltage of battery is connected to B
In forward biasing, the voltage at which the current will respectively be
starts to increase rapidly is known as cut-in or knee (a) 0.2 A and 0.1 A (b) 0.4 A and 0.2 A
voltage. For germanium it is 0.3 V while for silicon it (c) 0.1 A and 0.2 A (d) 0.2 A and 0.4 A
is 0.7 V. (JEE Main 2015)
dynamic resistance 4. In an unbiased n-p junction electrons diffuse from
It is defined as the ratio of a small change in voltage n-region to p-region because
DV applied across the p-n junction to a small change in (a) holes in p-region attract them
current DI through the junction. (b) electrons travel across the junction due to
rd = DV potential difference
DI
ideal diode (c) electron concentration in n-region is more as
A diode permits only unidirectional conduction. It compared to that in p-region
conducts well in the forward direction and poorly in (d) only electrons move from n to p region and not
the reverse direction. It would have been ideal if a diode the vice-versa
acts as a perfect conductor (with zero voltage across (JEE Main 2015)

34 Physics for you | march 16


rectifier
It is a device which converts ac voltage to dc voltage.
Rectifier is based on the fact that, a forward bias
p-n junction conducts and a reverse bias p-n junction
does not conduct. Rectifiers are of two types
Half wave rectifier
Full wave rectifier

half Wave rectifier Vm


Peak value of current is I m =
The circuit diagram, input and output voltage waveforms r + RL
f

for a half wave rectifier are as shown in the following 2I m


dc value of current is I dc =
figure.
I
rms value of current is I rms = m
2
Peak inverse voltage is P.I.V. = 2Vm
2I m
dc value of voltage is Vdc = I dc RL = R
L
ripple frequency
ur = ui = 50 Hz (half wave rectifier)
ur = 2ui = 100 Hz (full wave rectifier)
ripple factor
The ripple factor is a measure of purity of the dc output
of a rectifier, and is defined as
rms value of the components of wave
Peak value of current is r=
average or dc value
Vm
Im =
rf + RL I
2

where rf is the forward diode resistance, RL is the load r = rms 1


I dc
resistance and Vm is the peak value of the alternating For half wave rectifier,
voltage.
I I
Im I rms = m , I dc = m
rms value of current is I rms = 2
2 2
Im I /2
dc value of current is I dc = r = m 1 = 1.21
Im /
Peak inverse voltage is P.I.V. = Vm For full wave rectifier,
Im I 2I
dc value of voltage is Vdc = I dc RL = RL I rms = m , I dc = m
2
full Wave rectifier
2
The circuit diagram, input and output waveforms for a I / 2
r= m 1 = 0.483
full wave rectifier are as shown in the figure. 2I /
m

rectification Efficiency
The rectification efficiency tells us what percentage of
total input ac power is converted into useful dc output
power. Thus, rectification efficiency is defined as

Physics for you | march 16 35


dc power delivered to load different for each colour. The reverse breakdown
h= voltages of LEDs are very low, typically around 5 V.
ac input power from transformerr secondary
The semiconductor used for fabrication of visible LEDs
For a half wave rectifier,
must at least have a band gap of 1.8 eV. The compound
dc power delivered to the load is
2
semiconductor gallium arsenide phosphide (GaAsP)
2 I is used for making LEDs of different colours. GaAs is
Pdc = I dc RL = m RL
used for making infrared LED. The symbol of a LED is
Input ac power is shown in the figure.
2
2 I
Pac = I rms (r f + RL ) = m (r f + RL )
2
Rectification efficiency
P (I m / )2 RL 40.6 SELF CHECK
h = dc = 100% = %
Pac (I m / 2)2 (r f + RL ) 1 + r f / RL
5. The I-V characteristic of an LED is
If rf << RL, maximum rectification efficiency, h = 40.6%.
For a full wave rectifier,
dc power delivered to the load is
2
2 2I m
Pdc = I dc RL = R (a) (b)
L
Input ac power is
2
2 I
Pac = I rms (r f + RL ) = m (r f + RL )
2
Rectification efficiency (c) (d)
P (2I m / )2 RL
h = dc = 100%
Pac
( )
2
I m / 2 (r f + RL )
(JEE Main 2013)
81.2
= % Photodiode
1 + r f / RL
A photodiode is a special type p-n junction diode
If rf << RL, maximum rectification efficiency, h = 81.2%.
fabricated with a transparent window to allow light to
form factor fall on the diode. It is operated under reverse bias. When
I rms it is illuminated with light of photon energy greater
Form factor =
I dc than the energy gap of the semiconductor, electron-
For half wave rectifier, hole pairs are generated in near depletion region.
I /2 The symbol of a photodiode is shown in the figure
Form factor = m = = 1.57
Im / 2 below.
For full wave rectifier,
I / 2
Form factor = m = = 1.11
2I m / 2 2
solar cell
light Emitting diode (lEd) It converts solar energy into electrical energy. A solar
It converts electrical energy into light energy. It is a cell is basically a p-n junction which generates emf
heavily doped p-n junction which under forward bias when solar radiation falls on the p-n junction.
emits spontaneous radiation. The I-V characteristics It works on the same principle (photovoltaic effect) as
of a LED is similar to that of Si junction diode. But the photodiode, except that no external bias is applied
the threshold voltages are much higher and slightly and the junction area is kept large.

36 Physics for you | march 16


Zener diode thin. The emitter is heavily doped. The doping of the
It was invented by C. Zener. It is designed collector is between the heavy doping of the emitter and
to operate under reverse bias in the the light doping of the base. The function of the emitter
breakdown region and is used as a voltage is to emit or inject electrons (holes in case of a p-n-p
regulator. The symbol for Zener diode is transistor) into the base. The base passes most of these
shown in the figure. electrons (holes in case of p-n-p) onto the collector. The
Zener diode as a voltage regulator collector has the job of collecting or gathering these
The circuit diagram for Zener diode as a voltage electrons (holes in case of a p-n-p) from the base.
regulator is shown in the figure below. A transistor has two p-n junctions. One junction is
between the emitter and the base, and is called the
emitter-base junction, or simply the emitter junction.
The other junction is between the base and the collector,
and is called collector-base junction, or simply collector
junction.
The schematic representations of a n-p-n and p-n-p
KEY POINT transistors are shown in the figure.
The Zener diode is always reverse biased and this
reverse-bias voltage should be greater than the
breakdown voltage.

SELF CHECK
6. The value of the resistor, RS, needed in the dc voltage
regulator circuit shown here, equals The symbols for n-p-n and p-n-p transistors are shown
in the figure below.
RS
IL
Vi
nIL
VL

(a) (Vi VL)/n IL


(b) (Vi + VL)/n IL In operation of a transistor, IE = IB + IC
(c) (Vi VL)/(n + 1) IL where IE is emitter current, IB is base current, IC is the
(d) (Vi + VL)/(n + 1) IL collector current.
(JEE Main 2015)
A transistor can be operated in any one of the following
transistor
three configurations :
A transistor is basically a silicon or germanium crystal
Common emitter (CE)
containing three separate regions. It can either be
Common base (CB)
n-p-n-type or p-n-p-type. It has three regions. The middle
region is called the base and the two outer regions are Common collector (CC)
called the emitter and the collector. Although the two input characteristics of a transistor
outer regions are of the same type (n-type or p-type),
The variation of the input current with the input
their functions cannot be interchanged. The two regions
have different physical and electrical properties. In most voltage for a given output voltage is known as input
transistors, the collector region is made physically larger characteristics of a transistor.
than the emitter region since it is required to dissipate output characteristics of a transistor
more heat. The base is very lightly doped, and is very The variation of the output current with the output

Physics for you | march 16 37


voltage for a given input current is known as output where Ro and Ri are the output and input resistances.
characteristics of a transistor. Negative sign represents that output voltage is opposite
Action of a transistor in phase with the input voltage.
A transistor has two junctions-emitter junction and Power Gain
a collector junction. There are four possible ways of It is defined as the ratio of the output power to the input
biasing these two junctions as shown in the table. In power.
condition I, where emitter junction is forward biased
Output power (Po )
and collector junction is reverse biased. This condition Ap = = bac Av
is often described as forward reverse (FR). Input power (Pi )
V
Condition Emitter Collector Region of Note : Voltage gain (in dB) = 20 log10 o = 20 log10 Av
V
junction junction operation Po i
Power gain (in dB) = 10 log
I. FR Forward Reverse Active Pi
biased biased common base Amplifier
II. FF Forward Forward Saturation In common base transistor amplifier, the input signal
biased biased voltage and the output collector voltage are in the same
phase.
III. RR Reverse Reverse Cut off
biased biased dc Current Gain
IV. RF Reverse Forward Inverted It is defined as the ratio of collector current (IC) to the
biased biased emitter current (IE).
IC
transistor as a switch a dc =
IE
When the transistor is used in the cut off region or
saturation region, it acts as a switch. ac Current Gain
It is defined as the ratio of change in collector current
transistor as an Amplifier (DIC) to the change in emitter current (DIE).
When the transistor is used in the active region, it acts
as an amplifier. DI
a ac = C
DI E
common Emitter Amplifier
Voltage Gain
In the common emitter transistor amplifier, the input
signal voltage and the output collector voltage are 180 It is defined as the ratio of output voltage to the input
out of phase. voltage.
dc Current Gain Vo R
Av = = a ac o
It is defined as the ratio of the collector current (IC) to Vi Ri
the base current (IB). Power Gain
I It is defined as the ratio of output power to the input
b dc = C
IB power.
ac Current Gain output power (Po )
Ap = = aac Av
It is defined as ratio of change in collector current (DIC) input power (Pi )
to the change in base current (DIB). Relationship Between a and b
DI a
b ac = C b
DI B b= ;a =
1 a 1+b
Voltage Gain
It is defined as the ratio of output voltage to the input KEY POINT
voltage. Due to large value of the current amplification
V R factor, the CE configurations of transistor is
Av = o = b ac o preferred over the CB and CC configurations.
Vi Ri

38 Physics for you | march 16


transistor as an oscillator all the inputs are high.
A transistor can be used as an oscillator. The logic symbol of OR gate is
An oscillator generates ac output signal without any input
ac signal. An oscillator is a self sustained amplifier in
which a part of output is fed back to the input in the
same phase (called positive feedback).
The block diagram of an oscillator is shown in the figure.
The truth table for OR gate is

The Boolean expression for OR gate is


The circuit diagram of the tuned collector oscillator is Y=A+B
shown in the figure below. And gate
An AND gate has two or more inputs but only one
output. It is called AND gate because output is high
only when all the inputs are high.
The logic symbol of AND gate is

The truth table for AND gate is


The frequency of the oscillation is given by
1
u=
2 LC
logic gAtEs
A digital circuit with one or more input signals but only The Boolean expression for AND gate is
one output signal is known as logical gate. The logic Y = AB
gates are the basic building blocks of a digital system.
Each logic gate follows a certain logical relationship not gate
between input and output voltage. The NOT gate is the simplest of all logic gates. It has
There are three basic logic gates : only one input and one output. NOT gate is also called
OR gate inverter because it inverts the input.
AND gate The logic symbol of NOT gate is
NOT gate

truth table
It is a table that shows all possible input combinations The truth table for NOT gate is
and the corresponding output combinations for a logic
gate.
or gate
An OR gate has two or more inputs but only one output. The Boolean expression for NOT gate is
It is called OR gate because the output is high if any or Y=A

Physics for you | march 16 39


nAnd gate
It is an AND gate followed by a NOT gate.
The logic symbol for NAND gate is

The truth table for NAND gate is Y = AB = A + B = A + B


nor gate as a universal gate
NOR gate is called as universal gate because with the
repeated use of NOR gate we can construct any basic
gate.
The Boolean expression for NAND gate is NOT gate from NOR gate
Y = AB
nor gate
It is an OR gate followed by a NOT gate.
The logic symbol of NOR gate is Y=A
AND gate from NOR gate

The truth table for NOR gate is

Y = A + B = AB = AB
OR gate from NOR gate
The Boolean expression for NOR gate is
Y =A+B
nAnd as a universal gate
NAND gate is called as universal gate because with the
repeated use of NAND gate we can construct any basic Y =A+B=A+B
gate de Morgans theorems
NOT gate from NAND gate
A + B = AB

Y=A
AND gate from NAND gate NOR gate is equivalent to bubbled AND gate.
AB = A + B

Y = AB = AB
OR gate from NAND gate NAND is equivalent to bubbled OR gate.

40 Physics for you | march 16


Boolean identities (a) NAND gate (b) OR gate
A+B=B+A AB = BA (c) NOT gate (d) XOR gate
(AIEEE 2010)
A + (B + C) = (A + B) + C A(BC) = (AB)C
coMMunicAtion systEMs
A(B + C) = AB + AC A + BC = (A + B)(A + C)
The set up used for exchanging information between a
A+0=A A0 = 0
sender and receiver is called communication system.
A+1=1 A1 = A
A+A=A AA = A
A+ A=1 A A = 0

A=A A=A

A + B = AB AB = A + B
A communication system is composed of three basic units :
A + AB = A A(A + B) = A
Transmitter : The part of communication
A + AB = A + B A (A + B) = A B system, which sends out the information is called
transmitter.
SELF CHECK Transmission channel : The medium or the link,
which transfer message signal from the transmitter
7. Truth table for system of four NAND gates as shown
to the receiver of communication channel.
in figure is
Receiver : The part of the communication system,
which pick up the information sent out by the
transmitter is called receiver.
signal
A processed information converted into electrical pulse
for transmission is called a signal.
There are two forms of signal :
A B Y A B Y
Analog signal : A signal, which is a continuous
0 0 0 0 0 1 function of time. In an analog signal, current or
(a) 0 1 0 (b) 0 1 1 voltage value varies continuously with time.
1 0 1 1 0 0 Digital signal : A signal, which is a discontinuous
function of time, which has only two levels (either
1 1 1 1 1 0
low or high) is called a digital signal.
Transducer : A device which converts energy in
A B Y A B Y one form to another is called a transducer.
0 0 1 0 0 0
Bandwidth of signals
(c) 0 1 0 (d) 0 1 1
The frequency range of a signals is called its bandwidth.
1 0 0 1 0 1 Different types of signals require different ranges of
1 1 1 1 1 0 frequencies for proper communication.
These ranges of frequencies are called bandwidths of
(AIEEE 2012) the corresponding signals.
8. The combination of gates shown below yields Speech signals Bandwidth is 2800 Hz.
(from 300 Hz to 3100 Hz)
For music Bandwidth is 20 kHz.
Video signals Bandwidth is about 4.2 MHz.

Bandwidth of transmission Medium


For co-axial cable, bandwidth offered is 750 MHz,

Physics for you | march 16 41


(Normally operated below 18 GHz) wave is varied in accordance with the amplitude of the
For free space communication, audio frequency modulating signal. The frequency of
Standard AM broadcast (~ 540-1600 kHz) amplitude modulated wave remains unchanged as that
FM broadcast (~ 88-108 MHz) of the carrier wave.
Television (~ 54-890 MHz) If modulating signal m(t) = Am sinwmt ... (i)
Satellite communication
and carrier wave c(t) = Ac sinwct ... (ii)
(5.925-6.425 GHz for Uplink) where wc = angular frequency of carrier wave.
(3.7-4.2 GHz for Downlink) wm = angular frequency of modulating signal.
Modulation Am, Ac = Peak amplitudes of modulating and carrier
Modulation process is used for transmission of signal waves respectively.
A
from transmitter to receiver. In modulation, a high m = m (amplitude modulation index), m 1 or else
frequency carrier wave is used to carry the information Ac
signal over a long distance. distortion occurs.
The process of changing some characteristics e.g.
amplitude, frequency or phase of carrier wave in
accordance with instantaneous value of modulating
signal is known as modulation.

need of Modulation
The sound waves within the range of human
hearing have frequency range from 20 Hz to
20 kHz. The sound waves cannot be transmitted from
a radio transmitter by converting them into electrical
waves (audio signal) directly for the following reasons;
For efficient transmission and reception, the
transmitting and receiving antenna must have a
length equal to quarter wavelength of the audio
signal. For a frequency of 15 kHz of audio signal,
the length of the antenna comes out to be of the
order of 5000 m. To set up a vertical antenna of this
size is practically impossible.
The energy radiated from an antenna is practically
The amplitude modulated signal given by
zero, when the frequency of the signal to be
transmitted is below 15 kHz. It also makes the direct A
cm (t) = Ac sin w c t + m c cos(w c w m )t
transmission of audio signal as impractical. 2
mA
Due to the fact that all audio signals from different
c cos(w c + w m )t
2
sources possess frequencies in the same range This modulated wave contains frequencies
i.e., 20 Hz to 20 kHz, an audio signal cannot (wc wm), wc and (wc + wm).
be transmitted directly. It is because, the audio wc wm and wc + wm are called the lower side band and
signal from different transmitting stations will get upper side band frequencies respectively.
hopelessly and inseparably mixed up. In amplitude modulation (AM):
The aforementioned difficulties faced during the A A Amin
transmission of audio signal, if transmitted directly, are Index of AM, m = m = max
Ac Amax + Amin
overcome by the process of modulation.
Ac = Peak amplitude of high frequency carrier wave.
The following are three types of modulation.
Am = Peak amplitude of low frequency modulating
Amplitude Modulation signal.
In amplitude modulation the amplitude of carrier Required bandwidth = (wc + wm) (wc wm) = 2wm

42 Physics for you | march 16


For detection of AM wave, essential condition is
1
<< RC where RC = time constant of the circuit.
fc
The amplitude modulated signal contains d.c component
and also some different frequencies which are not
required. This signal is made to pass through a band
pass filter which rejects the d.c. components and also The frequency of modulated signal varies between
low and high frequencies. It allows a band of frequencies k f Em k f Em
f min = f c and f max = f c +
wc wm, wc, wc + wm to pass through. 2 2
where kf is the proportionality constant and determines
the maximum variation in frequency of modulated
wave for a given modulating signal.
The maximum swing of frequency of modulated wave
from the carrier frequency is called frequency deviation (d).
d = fmax fc = fc fmin
The modulation index (mf) of the frequency modulating
The frequency spectrum of amplitude modulated wave wave is
is shown in above figure. The two side band frequencies d f f f f
have equal amplitude (= mAc/2) which never exceeds mf = = max c = c min
fm fm fm
half the carrier amplitude.
The amplitude modulation index (m) define the quality Bandwidth of FM : In frequency modulated signal,
of the transmitted signal. When modulation index (m) the audio signal is contained in the sidebands. Since
is small, variation in carrier amplitude will not be large, the sidebands are separated from each other by the
therefore, audio signal being transmitted will not be frequency of modulating signal (fm),
strong. As the modulation index (m) increases but less bandwidth = 2n frequency of modulating signal
than 1, the audio signal on reception becomes more clear. where n is the number of significant sidebands pairs.
frequency Modulation Advantages of frequency modulation : The following
In frequency modulation, the frequency of carrier wave are a few advantages of frequency modulation over
is varied in accordance with the audio frequency signal. amplitude modulation.
The instantaneous values of the voltage of carrier waves FM reception is quite immune to noise as
and modulating signal can be represented as compared to AM reception. Noise is a form of
ec = Ec coswct ... (i) amplitude variations in the transmitted signal due
and em = Emcoswmt ... (ii) to atmosphere, industries, etc. In FM receivers, the
where ec, Ec, wc are the instantaneous value, peak value, noise can be reduced by increasing the frequency
angular frequency of the carrier and em, Em and wm are deviation or by making use of amplitude limiters.
the instantaneous value, peak value and the angular FM transmission is highly efficient as compared to
frequency of the modulating signal. AM transmission.
In FM transmission, all the transmitted power is
useful; whereas in AM transmission, most of the
power goes waste in the transmitted carrier, which
contains no useful information.
Due to a large number of sidebands, FM transmission
can be used for the stereo sound transmission.
Disadvantages : The following are a few disadvantages
of FM transmission:
The bandwidth in FM transmission is about 10 times
as large as that needed in AM transmission. As a

Physics for you | march 16 43


result, much wider frequency channel is required distant place via telephone line is known as facsimilie
in FM transmission. or FAX.
FM reception is limited to line-of-sight. Due to this, Earths Atmosphere
area of reception for FM is much smaller than that The gaseous envelope surrounding the earth is called
for AM. earths atmosphere. Earths atmosphere mainly consists
FM transmitting and receiving equipments are very of nitrogen 78%, oxygen 21% along with a littile portion
complex as compared to those employed in AM of argon, carbon dioxide, water vapour, hydrocarbons,
transmission. sulphur compounds and dust particles.
Earths atmosphere helps in the propagation of
SELF CHECK
electromagnetic waves from one place to another
9. A signal of 5 kHz frequency is amplitude modulated place. The various regions of earths atmosphere are as
on a carrier wave of frequency 2 MHz. The frequencies follows:
of the resultant signal is/are Troposphere : It extends upto a height of
(a) 2005 kHz, 2000 kHz and 1995 kHz 10 km, over earths surface.
(b) 2000 kHz and 1995 kHz Stratosphere : It extends from 10 km to 50 km.
(c) 2 MHz only There is an ozone layer in stratosphere which mostly
(d) 2005 kHz and 1995 kHz absorbs high energy radiations like ultraviolet
(JEE Main 2015) radiations etc. coming from outer space.
Mesophere : It extends from 50 km to 65 km.
demodulation
Ionosphere : It extends from 65 km to 400 km.
The process of detection or demodulation is the inverse
In this region, the temperature rises to some extent
process of modulation. In the process of demodulation,
with height, hence it is called thermosphere. The
audio signal is separated from the modulated signal.
ionosphere which is composed of ionised matter
Modem and fax (i.e., electrons and positive ions) plays an important
Modem is contraction of the term modulator and role in space communication. The ionosphere is
demodulator. A modem acts as a modulator in subdivided into four main layers as D, E, F1 and F2
transmitting mode and as a demodulator in receiving as shown in the table.
mode. The electronic transmission of a document at a
Name of the Approximate height Exists during Frequencies most affected
layer over earths surface
D 65-75 km Day only Reflects LF, absorbs MF and HF to some
degree
E 100 km Day only Helps surface waves, reflects HF
F1 170-190 km Day time, merges with Partially absorbs HF waves yet allowing them
F2 at night to reach F2.
F2 300 km at night, Day and night Efficiently reflects HF waves, particularly at
250-400 km during night
day time

radio waves
The electromagnetic waves of frequency ranging from
a few kilohertz to few hundred megahertz are called
radio waves..
Frequency range and wavelength range of radiowaves
are given in the table.

44 Physics for you | march 16


S. Frequency band Frequency range Wavelength Main use
No. range
1. Very-low frequency (VLF) 3 kHz to 30 kHz 10 km to 100 km Long distance point to point
communication
2. Low frequency (LF) 30 kHz to 300 kHz 1 km to 10 km Marine and navigational
purpose
3. Medium frequency (MF) 300 kHz to 3 MHz 100 m to 1 km Marine and broadcasting
purposes
4. High frequency (HF) 3 MHz to 30 MHz 10 m to 100 m Communications of all types
5. Very high frequency (VHF) 30 MHz to 300 MHz 1 m to 10 m T.V., Radar and air navigation
6. Ultra-high frequency (UHF) 300 MHz to 3000 MHz 10 cm to 1 m Radar and microwave
communication
7. Super-high frequency (SHF) 3 GHz to 30 GHz 1 cm to 10 cm Radar, radio relays and
navigation purposes
8. Extremely high frequency 30 GHz to 300 GHz 1 mm to 1 cm Optical fibre
(EHF) communication

space communication mesosphere and part of stratosphere) extends from about


The term space communication refers to sending 65 km to 400 km above the earths surface. Constituent
receiving and processing of information through space. gases are ionised in it by solar radiation. Through out
There are three types of space communication. the ionosphere, there are several layers in which the
ionisation density either reaches a maximum or remains
ground wave propagation roughly constant. These regions are designated as
This mode of propagation can exist when the D(65-75 km), E(100 km) and F(130 km-400 km) in
transmitting and receiving antenna are close to the order of approximate heights above earths surface.
surface of the earth. For radiating high efficiency signals, During day time the F layer splits into separate layers
the size of the antenna should be of the order of l/4. called F1 (170-190 km over earths surface) and F2 (250-
l = wavelength of the signal. 400 km over earths surface). During night F1 layer
The field component of such a launched wave soon usually disappears.
becomes vertically polarised as it glides over the surface
of the earth. The electrical fields due to the wave induce
charges on the earths surface. The ground wave is
weakened as a result of energy absorbed by the earth
during its propagation. These losses make ground waves
unsuitable for very long range communication.
Ground wave propagation can be sustained only at
low frequencies (~ 500 kHz to 1500 kHz) or for radio
broadcast at long wavelengths.
KEY POINT
The ground wave propagation is generally used for
local band broadcasting and is commonly called
medium wave. The maximum range of ground or
surface wave propagation depends on : Region of high conductivity is confined to a relatively
frequency of the radio waves and
thin layer at the lower edge of E-region and to the
power of the transmitter.
upper part of D-region. Therefore high frequencies
sky wave propagation (ionospheric propagation) are attenuated when they penetrate this region. D and
A transmitted wave going up is reflected back by the E layer disappears during night and low and medium
ionosphere which forms an ionised layer of electrons frequency communication also becomes possible.
and ions around the earth. The ionosphere (including Ionisation density increases as we go up from D layer to

Physics for you | march 16 45


F layer and then decreases, due to which their respective space wave propagation (tropospherical
refractive indices vary. Therefore the transmitted wave
propagation)
is reflected back in a way similar to the phenomenon of
total internal refraction. This mode is also known as line of sight communication.
1/2 To send signals at far away stations, either repeater
Ne 2 transmitting stations are necessary or height of the
where m = m 0 1 2
e 0mw transmitter is increased by locating it in a satellite. If
m = refractive index of particular region of ionospere, hT is the height of transmitting antenna and dT is the
m is the mass of the electron, N is the electron density in distance to the horizon from it, then dT = 2RhT ,
the ionosphere, e0 and n0 are permittivity and refractive where R is the radius of the earth. dT is called radio
index respectively for free space, e = electronic charge. horizon of transmitting antenna.
Ionosphere behaves as a rarer medium by which carrier The maximum line-of-sight distance dM between the
wave is reflected back if its frequency two antennas having heights hT and hR above the earth
(f fc) where fc = critical frequency. is given by
fc 9 (Nmax)1/2 d M = 2RhT + 2RhR
Nmax = maximum electron density of the ionosphere. fc
is called critical frequency and it represents the highest SELF CHECK
frequency that will be reflected back from a particular
layer at vertical incidence. 10. A radar has a power of 1 kW and is operating at a
Maximum usable frequency (MUF) : It is the highest frequency of 10 GHz. It is located on a mountain
frequency that can be reflected from a particular layer top of height 500 m. The maximum distance upto
of ionosphere for a given angle of incidence (i) is given which it can detect object located on the surface of
as MUF = fc sec i. the earth (Radius of earth = 6.4 106 m) is
KEY POINT (a) 16 km (b) 40 km (c) 64 km (d) 80 km
(AIEEE 2012)
The distance between the transmitting antenna
and point where the sky wave is first received after AnsWEr KEys (sElf chEcK)
returning to earth is called the skip distance.
1. (b) 2. (b) 3. (b) 4. (c) 5. (b)
Skip zone is a silent zone where no signal can be
picked up. 6. (c) 7. (d) 8. (b) 9. (a) 10. (d)
nn

Science Behind The JuncTion LaSer


In the arrangement of figure, there
are many electrons in the conduction
band of the n-type material and
can release its energy as a photon. This photon can stimulate a second
electron to fall into the valence band, producing a second photon by
stimulated emission. In this way, if the current through the junction
many holes in the valence band is great enough, a chain reaction of stimulated emission events can
of the p-type material. Thus, occur and laser light can be generated. To bring this about, opposite
there is a population faces of the p-n junction crystal must be flat and parallel, so that light
inversion for the can be reflected back and forth within the crystal. Thus, a p-n junction
electrons; that can act as a junction laser, its light output being highly coherent and
is, there are much more sharply defined in wavelength than light from an LED.
more electrons in Junction lasers are built into compact disc (cD) players, where, by
higher energy levels detecting reflections from the rotating disc, they are used to translate
than in lower energy microscopic pits in the disc into sound. They are also much used in
levels. This is normally a optical communication systems based on optical fibres. Junction
necessary - but not a sufficient lasers are usually designed to operate in the infrared region of the
- condition for laser action. electromagnetic spectrum because optical fibers have two windows in
When a single electron moves from the that region (at l = 1.31 and 1.55 mm) for which the energy absorption
conduction band to the valence band, it per unit length of the fiber is a minimum.

52 Physics for you | march 16


1. The input resistance of a common emitter transistor 5. What would be the percentage power saving if the
amplifier, if the output resistance is 500 kW, the carrier and one of the side bands were suppressed
current gain, a = 0.98, and power gain is 6.0625 106, in AM wave modulated to a depth of 75% before
is transmission took place?
(a) 200 W (b) 300 W (a) 89.02% (b) 9.1%
(c) 100 W (d) 400 W (c) 7.81% (d) 100%
2. For the transistor amplifier circuit shown below, value 6. Frequencies higher than 10 MHz were found not
of collector bias voltage VC should be approximately being reflected by the ionosphere on a particular
(Take VBE = 0.7 V) day at a place. The maximum electron density of the
ionosphere on that day was near to
(a) 1.5 1010 m3 (b) 1.24 1012 m3
12 3
(c) 3 10 m (d) None of these
7. The simplified output Y of the given logic circuit is

A
Y
B

(a) 6 V (b) 8 V
(c) 10 V (d) 11 V
3. The circuit shown in the figure contains three (a) A B + A B 3 (b) A B + A B
diodes each with forward resistance of 50 W (c) A B + A B (d) A B + A B
and with infinite backward resistance. If the 8. A sky wave with a frequency of 55 MHz is incident
battery voltage is 6 V, the current through the on the D-region of earths atmosphere at 30. The
100 W resistance is angle of refraction is (electron density for D-region
150 is 400 electron cc1)
50
(a) 60 (b) 15
(c) 45 (d) 30
50
9. A 24 V, 600 mW zener diode is used to provide a 24 V
100 stabilized supply to a variable load RL, as shown in
6V the figure. The value of the resistance R is
(a) 0 (b) 36 mA
(c) 43 mA (d) 50 mA
4. For a CE-transistor amplifier, audio signal voltage
across the collector resistance of 2 kW is 2V. Suppose
the current amplification factor of the transistor is (a) 320 W (b) 640 W
100. Find the input signal voltage and base current, (c) 960 W (d) 1280 W
if the base resistance is 1 kW. 10. The diode used in the circuit shown in the figure
(a) 0.01 V and 10 mA has a constant voltage drop of 0.5 V at all currents
(b) 0.1 V and 14 mA and a maximum power rating of 100 mW. What
(c) 0.001 V and 100 mA should be the value of the resistance R connected in
(d) 2.0 V and 10 mA series with diode for obtaining maximum current?

Physics for you | MARCH 16 53


(a) 2.5 W R 0.5 V 16. The logic circuit shown in figure (i) yields the truth
(b) 10 W table figure (ii). What is the gate X in the diagram?
(c) 12.5 W 2.5 V
(d) 15 W
11. The ratio of electron and hole currents in a semi-
7
conductor is and the ratio of drift velocities
4
5
of electrons and holes is , then the ratio of (a) OR gate (b) AND gate
4
concentration of electrons and holes will be (c) NAND gate (d) NOR gate
5 7 17. A Si and a Ge diode has identical physical dimensions.
(a) (b) The band gap in Si is larger than that in Ge. An
7 5
25 49 identical reverse bias is applied across the diodes.
(c) (d) (a) The reverse current in Ge is larger than that in Si.
49 25 (b) The reverse current in Si is larger than that in Ge.
12. In bridge rectifier circuit, as shown in figure, the (c) The reverse current is identical in the two
input signal should be connected between diodes.
(a) A and D (d) The relative magnitude of the reverse currents
cannot be determined from the given data
(b) B and C only.
(c) A and C 18. What is the base resistance RB in the circuit as
(d) B and D shown in figure, if hFE = 90?
13. The equivalent resistance of the circuit as shown in
figure, across AB is given by (Assume the diodes to
be ideal.)

(a) 29 kW (b) 82 kW
(c) 108 kW (d) 55 kW
(a) 6.24 W (b) 5.64 W 19. A ground receiver station is receiving a signal at
(c) 8.24 W (d) 5.64 W or 8.24 W 5 MHz, transmitted from a ground transmitter
at a height of 300 m located at a distance of
14. In an npn transistor, 1010 electrons enter the emitter
100 km. Identify the mode of propagation. (Radius of
in 106 s. 4% of the electrons are lost in the base. earth = 6.4 106 m; Nmax of ionosphere = 1012 m 3)
The current transfer ratio will be (a) ground wave propagation
(a) 0.98 (b) 0.97 (b) space wave propagation
(c) 0.96 (d) 0.94 (c) sky wave propagation
15. The following configuration of gates is equivalent to (d) satellite communication
G1
A 20. The output of the given circuit in figure
B Y
G3
G2

(a) NAND (b) XOR


(c) OR (d) AND

54 Physics for you | MARCH 16


(a) would be zero at all times 26. When a p-n junction diode made from germanium
(b) would be like a half-wave rectifier with positive or silicon is forward-biased, energy is released at
cycle in output the junction due to the recombination of electrons
(c) would be like a half-wave rectifier with negative and holes. This energy is in
cycle in output (a) Visible region (b) Infrared region
(d) would be like that of a full-wave rectifier. (c) UV region (d) X-ray region
21. A basic communication system consists of 27. A block of pure silicon at 300 K has a length of
(A) transmitter (B) information source 10 cm and an area of 1.0 cm2. A battery of emf 2 V
(C) user of information (D) channel is connected across it. The mobility of electron is
(E) receiver 0.14 m2 V1 s1 and their density is 1.5 1016 m3.
Choose the correct sequence in which these are The mobility of holes is 0.05 m2 V1 s1. The hole
arranged in a basic communication system. current is
(a) ABCDE (b) BADEC
(a) 2.0 107A (b) 2.2 107A
(c) BDACE (d) BEADC 7
(c) 2.4 10 A (d) 2.6 107A
22. A light beam entering an optical fibre makes an
28. The saturation current of a p-n junction diode made
angle of 10 with the fibre core-fibre clad boundary
surface. If the fibre core and clad refractive indices from germanium at 27C is 105 A. What will be
are 1.5 and 1.49 respectively, can this beam propagate required potential in order to obtain a current of
along the fibre? 250 mA in forward bias?
(a) No (Boltzmann constant, k = 1.38 1023 J K1).
(b) Yes (a) 0.26 V (b) 2.6 V
(c) Refractive indices have nothing to do with beam (c) 0.52 V (d) 1.04 V
propagation 29. The junction diode in the following circuit requires a
(d) The given data is insufficient minimum current of 1 mA to be above the knee point
23. An audio signal is modulated by a carrier wave (0.7 V) of its I-V characteristic curve. The voltage across
of 20 MHz such that the bandwidth required the junction diode is independent of current above the
for modulation is 3 kHz. If R = 10 kW and knee point, if VB = 4 V, then the maximum value of R
C = 0.01 mF, can the wave be demodulated? so that the voltage is above knee point will be
(a) Yes
(b) No
(c) Can not be predicted
(d) Data insufficient
24. Select the output Y of the combination of gates as
shown in figure for inputs A = 1, B = 0; A = 1, B = 1
and A = 0, B = 0 respectively.
(a) 3.3 kW (b) 4.0 kW
(c) 4.7 kW (d) 6.6 kW
30. Match the column I with column II and select the
correct option.
Column I Column II
(a) (0, 1, 1) (b) (1, 0, 1)
(c) (1, 1, 1) (d) (1, 0, 0) P. Microphone 1. It converts pressure
variation into
25. A 50 MHz sky wave takes 4.04 ms to reach a receiver
electrical signals.
via re-transmission from a satellite 600 km above
earths surface. Assuming re-transmission time by Q. Piezo-electric 2. It is the loss of strength
satellite negligible, find the distance between source sensor of signal during its
and receiver. propagation through
(a) 606 km (b) 170 km communication
(c) 340 km (d) 280 km channel.

Physics for you | MARCH 16 55


R. Photo-detector 3. It converts speech 150

signal into electrical D1


signals. 50

S. Attenuation 4. It converts light D2


signals into electrical
signals. 6V 100

P Q R S 1 1 1
= +
(a) 1 2 3 4 R r f + 150 W r f + 50 W
(b) 2 3 4 1 1 1
= +
(c) 3 4 1 2 50 W + 150 W 50 W + 50 W
(d) 3 1 4 2 1 1 1 200
= + or R = W
SolutionS R 200 W 100 W 3
R Total resistance of the circuit,
1. (a) : Voltage gain, AV = b 0
Ri 200 500
R = 100 W + W= W
a 0.98 3 3
Also current gain, b = = = 49 Current in the circuit,
1 a 1 0.98
V 6V
500 103 I= = = 3.6 102 A
AV = 49 R 500
Ri W
3
500 103 = 36 103 A = 36 mA
Power gain = bAV = 492 ...(i) 4. (a) : Given, collector resistance
Ri
Routput = 2 kW = 2000 W
and power gain = 6.0625 106 ...(ii) Current amplification factor of the transistor
\ From (i) and (ii), we get bAC = 100
49 49 500 103 Audio signal voltage, Voutput = 2 V
= 6.0625 106
Ri Input (base) resistance, Rinput = 1 kW = 1000 W
V R
49 5 105 49 Q Voltage gain, AV = output = b AC output
Ri = Vinput Rinput
6.0625 106
\ Input signal voltage
\ Ri = 198 W 200 W Voutput
V VBE (18 0.7) Vinput =
2. (c) : I B = CC = = 17.3 m A b AC (Routput / Rinput )
RB 106
Also, IC = b IB = 100 17.3 mA = 1.73 mA 2
VRC = ICRC = 1.73 mA 4.7 kW = 8.1 V = = 0.01 V
100 (2000 / 1000)
VC = VCC VRC = (18 8.1) = 9.9 V 10 V
So, the correct option is (c). Vinput 0.01
Base (input) current, I B = =
3. (b) : In the given circuit, the upper two diodes Rinput 1000
D1 and D2 are forward biased and the lower diode D3 = 10 106 A = 10 mA
is reverse biased. Hence, no current flows through
75 3
the lower diode D3. The equivalent circuit is as 5. (a) : Here, modulation index, m = =
shown in the figure. 100 4
As diode D1 and 150 W resistor are in series and Power produced by the AM transmitter
their combination is in parallel with the series
combination of diode D2 and 50 W resistor. Their m2 9 41
Pt = Pc 1 + = Pc 1 + = Pc
equivalent resistance is 2 32 32

56 Physics for you | MARCH 16


8. (d) : For D-region, f = 55 106 Hz , i = 30
m2 9 9
PSB = Pc = Pc = P N = 400 106 m3
4 16 4 64 c
81.45 N
On suppressing, power saved is m = 1
9 73 f2
= Pc + Pc = P
64 64 c
81.45 400 106 sin i
73 m = 1 1 \ m = =1
Pc (55 106 )2 sin r
\ Percentage saving = 64 100 or sin i = sin r
73 Pt
or i = r = 30
Pc
73 32
= 64 100 = 100 = 89.02% 9. (a) : Given, VZ = 24 V, PZ = 600 mW
41 64 41 Current through zener diode
Pc
32 3
PZ 600 10 W
IZ = = = 25 mA
80.6 N VZ 24 V
6. (b) : f 2 =
cos2 i Voltage drop across R = 32 V 24 V = 8 V
where i is angle of incidence and N is electron density. For 8V
\ R= = 320 W
the wave not being reflected from ionosphere , i = 0. 25 mA
80.6 N
\ f2 = = 80.6 N 10. (b) : Voltage drop across diode, Vd = 0.5 V
cos2 0 Maximum power rating, P = 100 mW
f2 (10 106 )2 = 100 103 W = 0.1 W
or N = = 2
80.6 80.6 V 2 (0.5 V)
Diode resistance, Rd = d = = 2.5 W
100 P 0.1 W
= 1012 = 1.24 1012 m3
80.6 Vd 0.5 V
Current in the diode, Id = = = 0.2 A
Y Rd 2.5 W
2.5 V
A Y Total resistance of the circuit = = 12.5 W
7. (a) : B
Y 0.2 A
Y Resistance R in circuit = 12.5 W 2.5 W = 10 W
11. (b) : I = nAevd or I nvd
Y = A B = A + B I nv
\ e = e e
I h nhvh
Y = A ( A + B ) = A + ( A + B )
= A + ( A B) = A + ( A B) ne Ie vh 7 4 7
or = = =
nh I h ve 4 5 5
Y = B ( A + B ) = B + ( A + B )
= B + ( A B ) = B + ( A B) 12. (d) : The input signal should be connected between
two points of bridge rectifier such that in positive
Y = [ A + ( A B)] [ B + ( A B)]
half wave of input signal, one p-n junction should
= [ A + ( A B)] + [ B + ( A B)] be forward biased and other should be reverse
= A ( A B) + B ( A B) biased and in negative half wave of input signal, the
reverse should take place. It will be so when input
= A B ( A + B ) = ( A + B )( A + B) is connected between B and D.
= A A + A B + A B + B B 13. (d) : When positive terminal of external battery is
= 0 + A B + A B + 0 connected to A and negative terminal of battery to B
( A A = 0 and B B = 0) then D1 is forward biased, will offer least resistance.
D2 is reverse biased, will offer maximum resistance.
= A B + A B The effective circuit will be as shown in figure (i).

Physics for you | MARCH 16 57


If IC is the collector current, then
5
9 4 = ICRC IC =
2 103
or IC = 2.5 mA
I 2.5 mA
IB = C = = 2.78 105 A
29 b 90
The effective resistance is+ 4 = 5.64 W .
2+9 Since the transistor operates in active region,
When negative terminal of external battery is therefore, VBE = 0.7 V
connected to A and positive terminal of battery to B, 3 0.7 2.3
\ RB = = 82 103 W
then D1 is reverse biased and D2 is forward biased. IB 2.78 105
The effective circuit will be as shown in figure (ii). = 82 kW
98
The effective resistance is 4 + = 8.24 W . 19. (c) : Maximum distance covered by space wave
9+8 communication
14. (c) : Number of electrons reaching the collector,
96 = 2RhT = 2 6.4 106 300 62 103 m.
nc = 1010 = 0.96 1010 Since receiver-transmitter distance is 100 km.
100
n e Therefore, signal cannot go either by ground wave
Emitter current, Ie = e or space wave.
t
nc e For sky wave, the value of critical frequency of
Collector current, Ic = signal should be
t I n 1 1
\ Current transfer ratio, a = c = c
Ie ne 12 2
fc = 9(N max )2 = 9(10 ) = 9 106 Hz = 9 MHz.
0.96 1010 As the signal frequency 5 MHz < 9 MHz, so the
= = 0.96
1010 communication is through sky waves.
15. (b) : Output of G1 = (A + B) 20. (c) : During positive half cycle of input ac voltage,
Output of G2 = A . B the p-n junction is forward biased. The resistance
Output of G3 is of p-n junction is low. The current in the circuit is
Y = ( A + B) . ( A . B) = ( A + B) . ( A + B ) maximum. In this situation, a maximum potential
difference will appear across resistance connected in
= A. A + A. B + B. A + B. B = A. B + A. B
series of circuit. Due to it, there is no output voltage
It is the Boolean of XOR gate. across p-n junction.
Hence, the given configuration of gates is equivalent During the negative half-cycle of input ac voltage,
to XOR gate. the p-n junction is reverse biased. The resistance of
16. (b) : From the truth table we note that, Y = A + B p-n junction becomes high which will be more than
i.e., it is OR gate. resistance in series. Due to it, there will be voltage
From gate in figure (i), Y = A + X = A + B. across p-n junction with negative cycle in output.
or A + X = A + B = A + B . ( A + A) 21. (b) : The block diagram of a communication system
is shown in the figure.
( A + A = 1)
= A + B . A + B . A = A . (1 + B) + A . B Information
Source
Transmitter Channel Receiver
User of
Information

= A + A. B 22. (a) : Critical angle of core-cladding surface is


So X = A . B , which is AND gate with input as A m 1.49
sin C = 2 = = 0.9933 = sin 8321
and B. Thus, gate X is AND gate. m1 1.50
17. (c) or C = 8321
18. (b) : Here, hFE = forward current ratio sin qa m
i.e. b = 90, VCE = 4 V, RC = 2 kW For optical fibre, = 1
sin(90 qi ) m 0

58 Physics for you | MARCH 16


sin qa m1 ct (3 108 )(4.04 103 )
or = \ x= =
cos qi m 0 2 2
m 1 = 6.06 105 m = 606 km
or cos qi = 0 sin qa = sin 10
m1 1.5 Let T be the source of
2 electromagnetic waves
= 0.1736 = 0.1157 (i.e. transmitter), R be
3
receiver and S be satellite
qi = 8320 at locations as shown in figure.
As the angle of incidence at the core and cladding d2 = x2 h2 = (606)2 (600)2 = 7236
surface is just equal to critical angle, so the ray will \ d = 85.06 km
not suffer total internal reflection. Hence, the beam Distance between source and receiver
cannot propagate along the fibre. = 2d = 2 85.06 170 km
23. (b) : fc = 20 MHz = 20 106 Hz 26. (b) : If p-n junction diode is made from germanium
1 1 or silicon, the energy released due to recombination
= 106 = 0.05 106 = 0.5 107s of free electrons and holes is in the infrared region.
fc 20 However, if the p-n junction diode is made from
Bandwidth = 2fm = 3 kHz = 3 103 Hz gallium arsenide or indium phosphide, then the
energy released due to recombination of free
fm = 1.5 103 Hz electrons and holes is in the visible region.
1 1 27. (c) : In a pure semiconductor,
= 103 = 0.7 103 s.
fm 1.5 ne = nh = 1.5 1016 m3
V 2
Diode detector can demodulate when E= = = 20 Vm1
1 1 l 10 102
<< RC <<
fc fm vh = mh E = 0.05 20 = 1.0 m s1
Ih = nh Aevh
Q RC = 10 kW 1 108 F = 104 s = (1.5 1016) (1.0 104) (1.6 1019) 1.0
It can be demodulated. = 2.4 107 A
24. (d) : 28. (a) : Here, I0 = 105 A, T = 27 + 273 = 300 K,
I = 250 103 A
I = I0 [eeV/kT 1]
I 250 103
or eeV / kT = +1 = + 1 = 25001
I0 105
eV 10.126 kT
or = ln(25001) = 10.126 or V =
kT e
23
10.126 1.38 10 300
= = 0.26 V .
19
1.6 10
29. (a) : As, VB = Vknee + IR
or4 = 0.7 + 103R
3.3
or R = = 3.3 103 W = 3.3 k W
3
10
30. (d)
nn
25. (b) : Here,
Total time taken, t = 4.04 ms = 4.04 103 s Common sense is
Let x be the distance of satellite from the surface of nothing more than a
earth. deposit of prejudices laid
Total time taken (t) = down by the mind before
you reach eighteen.
Total distance travelled (2x )
Albert Einstein
Speed of electromagnetic waves (c)

Physics for you | MARCH 16 59


chapterwise McQs for practice
Useful for All National and State Level Medical/Engg. Entrance Exams
current electricity 6. In cosmic rays 0.15 protons cm2 s1 are entering
1. The potential difference applied to an X-ray tube is the earths atmosphere. If the radius of the earth is
5 kV and the current through it is 3.2 mA. Then the 6400 km, the current received by the earth in the
number of electrons striking the target per second is form of cosmic rays is nearly
(a) 2 1016 (b) 5 106 (a) 0.12 A (b) 1.2 A
(c) 1 1017 (d) 4 1015 (c) 12 A (d) 120 A

2. In a hydrogen atom, an electron moves in an orbit 7. What is the equivalent


of radius 5.0 1011 m with a speed of 2.2 106 m s1. resistance across the
Find the equivalent current. points A and B in the
(Given, electronic charge = 1.6 1019 coulomb) given circuit?
(a) 1.12 103 A (b) 1.5 103 A (a) 8 W
6
(c) 3.2 10 A (d) 1.12 106 A (b) 12 W
(c) 16 W
3. Masses of the three wires of same material are in the
(d) 32 W
ratio of 1 : 2 : 3 and their lengths in the ratio 3 : 2 : 1.
Electrical resistance of these wires will be in the
ratio of 8. A ring is made of a wire having a resistance R0 = 12 W.
(a) 1 : 1 : 1 (b) 1 : 2 : 3 Find the points A and B as shown in the figure,
(c) 9 : 4 : 1 (d) 27 : 6 : 1 at which a current carrying conductor should be
connected so that the resistance R of the sub-circuit
4. The resistance of a wire at 300 K is found to be 0.3 W.
between these points is equal to 8/3 W.
If the temperature coefficient of resistance of wire is
1.5 103 K1, then the temperature at which the
resistance becomes 0.6 W, is
(a) 720 K (b) 345 K
(c) 993 K (d) 690 K
5. What is the drift velocity of electrons, if the current
flowing through a copper wire of 1 mm diameter is
1.1 A? Assume that, each atom of copper contributes
l1 5 l1 1
one electron. (Given, density of Cu = 9 g cm3 and (a) = (b) =
atomic weight of Cu = 63) l2 8 l2 3
(a) 0.3 mm s1 (b) 0.5 mm s1 l1 3 l1 1
(c) = (d) =
(c) 0.1 mm s1 (d) 0.2 mm s1 l2 8 l2 2

60 Physics for you | March 16


9. A cell having an emf E and internal resistance r, are 252 14
connected across a variable external resistance R. (a) 21 W (b) 7 W (c) W (d) W
85 3
As the resistance R is increased, the plot of potential
difference V across R is given by 13. In the figure, the
potential at points B
and C, and the value
(a) (b) of resistance R are
(a) 10 V, 4 V, 4 W
(b) 6 V, 4 V, 6 W
(c) 10 V, 4 V, 6 W
(c) (d) (d) 6 V, 4 V, 4 W
14. If the power dissipated
in the 9 W resistor in the
10. In the circuit given here, circuit shown is 36 W, then
the points A, B and C are the potential difference
at 70 V, zero and 10 V across the 2 W resistor is
respectively. Then,
(a) 8 V
(a) the point D will be at a
(b) 10 V
potential of 60 V
(b) the point D will be at a (c) 2 V
potential of 20 V (d) 4 V
(c) currents in the paths AD, DB and DC are in the 15. Three identical bulbs are connected in series and
ratio of 1 : 2 : 3 these together dissipate a power P. If the bulbs are
(d) currents in the paths AD, DB and DC are in the connected in parallel, then the power dissipated
ratio of 3 : 2 : 1 will be
11. A meter bridge is set-up as shown, to determine
(a) P/3 (b) 3P (c) 9P (d) P/9
an unknown resistance X using a standard 10 W
resistor. The galvanometer shows null point when moving charges and magnetism
tapping-key is at 52 cm mark. The end-corrections
16. An element D l = Dxi is placed at the origin and
are 1 cm and 2 cm respectively for the ends A and
B. The determined value of X is carries a large current I = 10 A. What is the magentic
field on the y-axis at a distance of 0.5 m if Dx = 1 cm?
(a) 4 108 T (b) 8 108 T
(c) 4 105 T (d) 8 105 T
17. In hydrogen atom, the electron is making
6.6 1015 rev s1 around the nucleus of radius 53 .
The magnetic field produced at the centre of the
(a) 10.2 W (b) 10.6 W (c) 10.8 W (d) 11.1 W orbit is nearly
(a) 0.12 Wb m2 (b) 1.2 Wb m2
12. A bridge circuit is shown in the figure. The
(c) 12 Wb m2 (d) 120 Wb m2
equivalent resistance between points A and B is
18. The magnetic field normal to the plane of a wire
of n turns and radius r which carries a current I is
measured on the axis of the coil at a small distance
h from the centre of the coil. This is smaller than the
magnetic field at the centre by the fraction
(a) (2/3)r2/h2 (b) (3/2)r2/h2
2 2
(c) (2/3)h /r (d) (3/2)h2/r2

Physics for you | March 16 61


19. Two parallel wires P and Q placed at a separation of (a) 103 Wb m2 (b) 105 Wb m2
r = 6 cm carry electric currents I1 = 5 A and I2 = 2 A (c) 1016 Wb m2 (d) 102 Wb m2
in opposite directions as shown in figure. Find the 24. A deuteron of kinetic energy 50 keV is describing a
point on the line PQ where the resultant magnetic circular orbit of radius 0.5 m, in a plane perpendicular
field is zero. to magnetic field B. The kinetic energy of a proton
that describes circular orbit of radius 0.5 m in the
same plane with the same magnetic field is
(a) 200 keV (b) 50 keV
(c) 100 keV (d) 25 keV
(a) 4 cm to the right of Q
(b) 9 cm to the left of P 25. A galvanometer of resistance 25 W is connected to
(c) 2 cm to the right of P a battery of 2 V along with a resistance in series.
(d) 3 cm to the left of Q When the value of this resistance is 3000 W, a
full scale deflection of 30 units is obtained in the
20. Two wires PQ and QR,
carry equal currents I as galvanometer. In order to reduce this deflection to
shown in figure. One end 20 units, the resistance in series will be
of both the wires extends (a) 4513 W (b) 5413 W
to infinity and PQR = q. (c) 2000 W (d) 6000 W
The magnitude of the 26. The current sensitivity of a moving coil galvanometer
magnetic field at O on the increases by 35 %, when its resistance is increased by
bisector angle of these a factor of 3. The voltage sensitivity of galvanometer
two wires at a distance r changes by a factor
from point Q is (a) 35% (b) 45%
m I q m I q (c) 55% (d) None of these
(a) 0 sin (b) 0 cot
4p r 2 4p r 2 27. A coil in the shape of an equilateral triangle of side
m I q m0 I (1 + cos q / 2) 0.02 m is suspended from its vertex such that it is
(c) 0 tan (d)
4p r 2 2 p r (sin q / 2) hanging in a vertical plane between the pole pieces
21. A solenoid of length 50 cm and radius of cross- of permanent magnet producing a uniform field of
section 1 cm has 1000 turns of wire wound over it. 5 102 T. If a current of 0.1 A is passed through
If the current carried is 5 A, the magnetic field on its the coil, what is the couple acting?
axis, near the centre of the solenoid is approximately 7 10
(permeability of free space m0 = 4p 107 T m A1) (a) 5 3 10 N m (b) 5 3 10 Nm
(a) 0.63 102 T (b) 1.26 102 T 3
(c) 107 N m (d) None of these
(c) 2.51 102 T (d) 6.3 T 5
22. A long straight wire of radius a carries a steady 28. Two thin long parallel wires separated by a distance
current I. The current is uniformly distributed b are carrying a current I ampere each. The
across its cross-section. The ratio of the magnetic magnitude of the force per unit length exerted by
a one wire on the other, is
field at and 2a is m0 I
2 m I2 m0 I m I2
(a) 1 : 4 (b) 4 : 1 (c) 1 : 1 (d) 1 : 2 (a) 02 (b) 2 (c) (d) 0
b 2 pb 2 pb 2 pb
23. A particle of charge 16 1018 coulomb moving 29. A galvanometer of resistance 100 W gives a full
with velocity 10 m s1 along the X-axis enters a scale deflection for a current of 105 A. To convert
region, where a magnetic field of induction B is it into an ammeter capable of measuring upto 1 A,
along the Y-axis and an electric field of magnitude we should connect a resistance of
104 V m1 is along the negative Z-axis. If the charged (a) 1 W in parallel (b) 103 W in parallel
particle continues moving along the X-axis, the (c) 105 W in series (d) 100 W in series
magnitude of B is
62 Physics for you | March 16
30. In a cyclotron, a magnetic induction of 1.4 T is 2 + 81 103 = 1 + 1.5 103t
used to accelerate protons. What should be the 2 + 0.081 = 1 + 1.5 103t
frequency of applied electric field? The mass and 1.081
t= = 720 C = 993K
charge of proton are 1.67 1027 kg and 1.6 1019 C 1.5 10 3
respectively. 5. (c) : As, I = neAvd
(a) 2.5 107 Hz (b) 2.14 107 Hz where, the number of electrons
(c) 3.5 107 Hz (d) 3.84 107 Hz Avogadros number
solutions n=
Volume of 63 g of copper
q ne
1. (a) : As I = = 6.02 1023 6.02 1023
t t = = cm 3
It 3.2 103 1 63 103 7
\ n= = = 2 1016
e 1.6 10 19 9 103
2. (a) : Here r = 5.0 1011 m, 6.02 1029 3
v = 2.2 106 m s1, e = 1.6 1019 C \ n= m
7
Period of revolution of electron, I
Also, vd =
2 pr 2 p 5.0 1011 neA
T= = s 1. 1 7
v 2.2 106 =
6.02 10 1.6 1019 p (0.5 103 )2
29
1 2.2 106
Frequency, u = = = 0.1 103 m s1 = 0.1 mm s1
T 2 p 5.0 1011
2.2 7 1017 6. (a) : Surface area of earth, A = 4pr2
= = 7 1015 s 1 Charge entering the earth per second per unit area
2 22 5
J = 0.15 1.6 1019 cm2s1
Current, I = eu = 1.6 1019 7 1015 = 0.15 1.6 1019 104 m2 s1
= 1.12 103 A
\ Current, I = J A = J 4pr2
3. (d) : Mass, M = volume density = Al d
= 0.15 1.6 1019 104 4 3.14 (6.4 106)2
or A = M/ld
= 0.12 A
rl 2d
Resistance, R = rl /A = rl /(M/ld) = 1 1 1 5 1
M 7. (a) : = + = = R1 = 2 W
So, R l2/M R1 10 2.5 10 2
l2 l2 l2 Now 2 W and 10 W are in series
Thus, R1 : R2 : R3 = 1 : 2 : 3
M1 M2 M3 R2 = 10 + 2 = 12 W
2 2 2 R2 and 12 W are in parallel
3 2 1
= : : = 27 : 6 : 1 1 1 1
1 2 3 = + R3 = 6 W
R3 12 12
4. (c) : Given, R300 = 0.3 W, Rt = 0.6 W
and T = 300 K = 27C Now, R3 and 10 W are in series
Temperature coefficient of resistance, R4 = 10 + 6 = 16 W
a = 1.5 103 K1 Now, R4 and 16 W are in parallel
\ R300 = R0(1 + a 27) 1 1 1
\ = + R=8W
0.3 = R0 (1 + 1.5 103 27) ...(i) R 16 16
Again, Rt = R0(1 + at) l
8. (d) : Resistance of wire, R = r
0.6 = R0(1 + 1.5 103 t) ...(ii) \ Rl
A
Dividing eq. (ii) and eq. (i), we get Here, R0 = R1 + R2 = 12 W ...(i)
0. 6 1 + 1.5 103 t R1 R2 8
= and = W (... R1 and R2 are in parallel)
0.3 1 + 1.5 103 27 R1 + R2 3
2(1 + 1.5 103 27) = 1 + 1.5 103t Putting R1 + R2 = 12 W in above equation, we get

Physics for you | March 16 63


R1R2 = 32 W ...(ii) \ VB 0 = 10 V or VB = 10 V
After solving eq. (i) and (ii), we get Potential difference between the points C and D
R1 = 4 W and R2 = 8 W VC VD = 1 4 = 4 V
l1 R1 1 \ VC 0 = 4 or VC = 4 V
Hence, = =
Potential difference between the point B and C
l2 R2 2
9. (c) : Here E = I(R + r) I = E/(R + r) 6
= 10 4 = 6 V \ R = = 6 W
Also, E = IR + Ir or E = V + Ir 1
Er E ER 14. (b) : The power dissipated in resistor of 9 W is
\ E =V + V = E r =
R+r R+r R+r V2 V2
P = 1 36 = 1 V12 = 36R
E R R
V=
1+ r / R V1 = 36 9 V1 = 6 3 = 18 V
So, graph must be as given in option(c). \ Total current through circuit is
10. (d) : Applying Kirchhoff s law at junction point D, 96 18
V = IReq 18 = I 18 = I I = 5 A
we get 9 + 6 5
I1 = I2 + I3 Potential across 2 W is
VA VD VD VB VD VC V2 = IR V2 = 5(2) = 10 V
= + 15 (c) : By Joules law, the power dissipated through a
10 20 30
resistor R, having a potential difference V is
VD VD 10
70 VD = + V2
2 3 P=
VD = 40 V R
When bulbs are connected in series, then
70 40
\ I1 = =3 A R = R + R + R = 3 R
10
V2
40 0 Power dissipated, P = ...(i)
I2 = =2A 3R
20 When they are connected in parallel, then
40 10 1 1 1 1 3 R
and I3 = =1 A = + + = R =
30 R R R R R 3
11. (b) : Using the condition for balanced Wheatstone 2
V
Power dissipated, P = ...(ii)
bridge, we get R/3
X (52 + 1)cm 53 From eq. (i) and (ii), we have
= =
10 W (100 52 + 2)cm 50 P=9P
53 10 16 (a) : Here dl = Dx = 1 cm = 102 m, I = 10 A,
or X= = 10.6 W r = y = 0.5 m, q = 90, m0/4p = 107 T m A1
50
12. (d) : The Wheatstone bridge is balanced, because
3W 6W
=
4W 8W
\ 7 W resistance is ineffective, we now have (3 W + 4 W)
and (6 W + 8 W) resistance in parallel.
7 14 14 According to Biot-Savart law,
\ RAB = = W
7 + 14 3
m0 Idl sin q 107 10 102 sin 90
13. (c) : Let the potential at points A, B, C and D be dB = =
VA, VB, VC and VD respectively. 4p r2 (0.5)2
Potential difference between the points B and A = 4 108 T

VB VA = 10 V The direction of the field dB will be the direction
As, earth potential is taken zero, so VA = 0

of vector dl r .
64 Physics for you | March 16


dl r = Dxi y j = Dxy(i j) = Dxyk m I
B = 0 [sin(90 q / 2) + sin 90] 2
4p a
Hence field dB is in the +z-direction.
m I m m I m I (1 + cos q / 2)
17. (a) : B = 0 = 0 (eu) = 0 (cos q / 2 + 1) = 0 .
2r 2r 2 p r sin q / 2 2 p r sin q / 2
4 p 107 1.6 1019 6.6 1015 21. (b) : The magnetic field is given by B = m0nI
= where, m0 = 4p 107 T m A1
2 53 1010
1000
= 0.12 Wb m2 n= ,I = 5 A
50 102
m 2 pnI m 2 pnIr 2 1000
18. (d) : B1 = 0 and B2 = 0 \ B = 4 p 107 5
4p r 4 p (r 2 + h2 )3/2 50 102
3/2
B h2 B = 1.26 102 T
So, 2 = 1 + 2
B1 r 22. (c) : Current density, J = I
Fractional decrease in the magnetic field will be pa2
B1 B2 B2
= 1
B1 B1
3/2
h2 = 1 1 3 h = 3 h
2 2
= 1 1 +
r 2 2 r2 2 r2

19. (a) : At the required point, the resultant magnetic


field will be zero when the fields due to the two
wires have equal magnitude and opposite directions.
Such point should lie either to the left of P or to From Amperes circuital law

the right of Q. But the wire Q has smaller current, B dl = m0 Ienclosed
the point should lie closer to the right of Q. Let
For r < a, B 2pr = m0 J pr2
this point be R at distance x from Q, as shown in
m I r
figure. B= 0
pa2 2
m I
At r = a/2, B1 = 0 ...(i)
4 pa
m I
Field due to current I1 at point R, For r > a, B 2pr = m0I B = 0
m I 2 pr
m0 I1 At r = 2a, B2 = 0 ...(ii)
B1 = , 4 pa
2 p(r + x ) B1
Field due to current I2 at point R, So, =1
B2
m I
B2 = 0 2 , 23. (a) : Given q = 16 1018 C, v = 10i m s
1
2 px
But B1 = B2 B = B j, E = 104 k Vm 1
I1 I
\ = 2 F e = qE = 16 1018 (104 k )
r+x x
14
or x = 2
I r
=
2 A 6 cm
= 4 cm = 16
10 kN
I1 I2 5A 2A F m = q(v B) = 16 1018 (10i B j)
20. (d) : Perpendicular to O from PQ or QR, a = r sin q/2 = 16 1017 B k N
Magnetic field induction at O due to current As the particle continues to move along the same
through PQ and QR is direction,

Physics for you | March 16 65


Fm = Fe
1 a 3
\ 16 1017 B = 16 1014 or A= a
or B = 103 Wb m2 2 2
24. (c) : In this case, magnetic force provides necessary 3a2 3 (0.02)2
= = = 3 104 m2
centripetal force i.e. 4 4
mv 2
qvB = t = 0.1 3 104 5 102 sin 90
r
mv 2mE = 5 3 107 N m
Radius of path, r = =
Bq qB
28. (d) : Let two long parallel thin wires X and Y carry
2mE 2m1E1 current I and separated by a distance b apart.
\ r= =
Bq Bq The magnitude of magnetic field B at any point on
mE (2m1 ) Y due to current I1 in X is given by
or E1 = = 50 keV [... m = 2m1] m I
m1 m1 B= 0 1
= 100 keV 2p b
The magnitude of force acting on length l of Y is
25. (a) : Current through galvanometer,
m I
V 2 F = I2 Bl = I2 0 1 l
Ig = = = k 30 ...(i) 2p b
R + G 3000 + 25
where k is figure of merit of galvanometer. The Force per unit length is
current corresponding to 20 units deflection of F m0 I1I2
=
galvanometer, l 2p b
Ig 2 2 2 Given I1 = I2 = I, therefore
I= 20 = I g = A
30 3 3 3025 F m0 I 2
=
If R is the resistance to be used in series of l 2p b
galvanometer, then 29. (b) : G = 100 W, Ig = 105 A, I = 1 A
V 2 2 2 To convert the galvanometer into an ammeter, we
I= or = .
R + G 3 3025 R + 25 should connect a resistance S in parallel to it.
On solving, we get \ Ig G = (I Ig) S
R = 4512.5 W 4513 W
Ig 105
35 135 S= G = 100
26. (c) : Given I s = I s + I = I I Ig 1 105
100 s 100 s
I 103
Initial voltage sensitivity, Vs = s or S = = 103 W
R 1 0.00001
30. (b) : Here, B = 1.4 T, m = 1.67 1027 kg,
I e = 1.6 1019 C
New voltage sensitivity, Vs = s
R The time required by a charged particle to complete
135 1 9 a semicircle in a dee is
= I = V
100 s 3R 20 s
pm 3.14 1.67 1027
t= = = 2.34 108 s.
% decrease in voltage sensitivity eB 1.6 10 19
1. 4
9
Vs Vs Thus, the direction of electric field should reverse
Vs Vs 20
V 100% = Vs
100% = 55%
after every 2.34 108 s.
s
The frequency of the applied electric field should be
27. (a) : Torque, t = IAB sin q, I = 0.1 A, q = 90 1 1
1 fc = = = 2.14 107 Hz.
A = base height 2t 2 2.34 108
2
nn
66 Physics for you | March 16
Exam on
22nd May

PRACTICE PAPER 2016


PaPer-I
section 1 (MaxiMuM Marks : 32) 4. Particle P1 moving with velocity 10 m s1 experienced
This section contains EIGHT questions a head-on-collision with a stationary particle P2
The answer to each question is a single digit integer of the same mass. As a result of the collision, the
ranging from 0 to 9, both inclusive kinetic energy of the system decreased by 50%. Find
1. Let us assume that air is under standard conditions the magnitude of the velocity of particle P1 after the
collision.
close to the Earths surface. Presuming that
the temperature and the molar mass of air are 5. A 1.0 g sample of pure KCl from the chemistry
independent of height, find the air pressure (in atm) stockroom is found to be radioactive and to decay
in a mine at the depth 5.0 km below the surface. at an absolute rate of 1600 counts s1. The decays are
(Molar mass of air is 28 g mol1) traced to the element potassium and in particular to
the isotope 40K, which constitutes 1.18% of normal
2. A thin convergent lens is placed between an object
potassium. The half-life for this decay is 13 10N
and a screen whose positions are fixed. There are
years. Find N. (Molar mass of KCl is 74.9 g mol1.)
two positions of the lens at which the sharp image
of the object is formed on the screen. Find the 6. A point isotropic source is located on a line
transverse dimension (in mm) of the object if at one perpendicular to the plane of a ring drawn through
position the transverse dimension of the image is the centre of the ring. The distance between the
centre of the ring and the source is 1 m, the radius
h1 = 8 mm and at the other h2 = 2 mm.
of the ring is 0.50 m. Find the rate of mean energy
3. One of the circuits for the measurement of flow (in W) across the area enclosed by ring if the
resistance by potentiometer is shown in the figure. intensity of source at the centre of ring is equal to
The galvanometer is connected at point A and zero I0 = 3 W m2.
deflection is observed at length PJ = 30 cm when
7. What is the maximum horizontal force F (in N)
es = 10 V and r = 1 W. In second case the secondary
that may be applied to the plank of mass 3/7 kg for
cell is changed as es = 5 V and r = 2 W, then zero which the solid sphere does not slip as it begins to
deflection is observed at length PJ = 10 cm. What roll on the plank. The sphere has a mass 0.5 kg and
is the current (in A) through resistance R when radius 5 cm. The coefficient of static and kinetic
PJ = 30 cm ? friction between the sphere and the plank are 0.25
and 0.24 respectively.
M

m F

Physics For you | march 16 67


8. Two long parallel wires of negligible resistance are 11. Consider a cycle followed by an engine, shown in
connected at one end to a resistance R and at other figure.
P
end to a dc voltage source. The distance between the
axes of the wire is 20 times greater than the cross- 1 2
sectional radius of each wire. The resultant force
of interaction between the wires turn into zero for
R = 3.6 10 p W. What is the value of p ? (Neglect
3
the effect of gravity.)
section 2 (MaxiMuM Marks : 40) V
This section contains TEN questions 1 to 2 is isothermal
Each question has FOUR options (a), (b), (c) and (d). 2 to 3 is adiabatic
ONE OR MORE THAN ONE of these four option(s) 3 to 1 is adiabatic
is(are) correct Such a process does not exist because
9. In two systems of units, the relation between velocity, (a) heat is completely converted into mechanical
energy, which is not possible
v e2
acceleration and force is v2 = 1 , a2 = a1et, and (b) mechanical energy is completely converted into
F1 t heat, which is not possible
F2 = , where e and t are constants. Relation (c) curves representing two adiabatic processes do
et
between mass, length and time in the two systems not intersect
is correctly given by (d) curves representing an adiabatic process and an
isothermal process do not intersect.
M1 L e2
(a) M2 = 2 2 (b) L2 = 1 12. A motorboat of mass m moves along a lake with
e t T2 velocity v0. At the moment t = 0 the engine of the
Te L e3 boat is shutdown. Assume the resistance of water
(c) T2 = 1 (d) L2 = 1
t2 T3 to be proportional to the velocity of the boat
10. In a photoelectric effect experiment, the maximum F = rv.
kinetic energy of the ejected photoelectrons is (a) The motorboat moved a distance mv0/r with
measured for various wavelengths of the incident the shutdown engine.
light. Figure shows a graph of this maximum kinetic mv0
(b) The motorboat moved a distance with the
energy Kmax as a function of the wavelength l of the shutdown engine. 2r
light falling on the surface of the metal. Which of (c) The velocity of the motorboat as a function of
the following statements is/are correct? time is v0e(r/m)t.
(d) The mean velocity of the motorboat over the
time interval (beginning with the moment
t = 0), during which its velocity decreases h
times is v0(h 1)/h lnh.
13. A charge particle of charge q and mass m is moving
with velocity v as shown in figure in a uniform
magnetic field B along negative z-direction. Select
the correct alternative(s).
y

(a) Threshold frequency for the metal is 1.2 1015 Hz.




(b) Work function of the metal is 4.968 eV. Extend upto a

(c) Maximum kinetic energy of photoelectrons large distance


corresponding to light of wavelength 100 nm is q, m


30

nearly 7.4 eV.
v x

(d) Photoelectric effect takes place with red light.

68 Physics For you | march 16


(a) Velocity of the particle when it comes out from n1 = 1 n2 = 1.5 n1 = 1

the magnetic field is v = v cos60 i + v sin60 j . n1 = 1
(b) Time for which the particle was in magnetic
m
field is .
3qB
mv
(c) Distance travelled in magnetic field is .
2qB S1 S2 S3 S4
(d) None of these.
(a) 25 cm left of S4, virtual
14. Water is flowing smoothly through a closed pipe
(b) 25 cm right of S4, virtual
system. At one point A, the speed of the water is
(c) 15 cm left of S4, virtual
3.0 m s1 while at another point B, 1.0 m higher, the
(d) 20 cm right of S4, virtual
speed is 4.0 m s1. The pressure at A is 20 kPa when
the water is flowing and 18 kPa when the water flow 17. A hemisphere of radius R is charged uniformly with
stops. Then the pressure at B when water surface density s. Then,
(a) is flowing is 6.7 kPa. sR
(a) electric potential at the centre is
(b) is flowing is 8.2 kPa. 2e 0
(c) stops flowing is 10.2 kPa. sR
(b) electric potential at the centre is
(d) stops flowing is 8.2 kPa. 4 e0
s
15. In figure, R is a fixed conducting ring of negligible (c) electric field at the centre is
4 e0
resistance and radius a. PQ is a uniform rod of
s
resistance r. It is hinged at the centre of the ring and (d) electric field at the centre is .
2e 0
rotated about this point in clockwise direction with
a uniform angular velocity w. There is a uniform 18. If, in a hydrogen atom, radius of nth Bohr orbit is rn,
magnetic field of strength B pointing inward and r and area enclosed by the nth orbit is An, then which
is a stationary resistance. Then, of the following graphs are correct?

rn
B rn log
r1
(a) (b)
P Q
n log n
r
R

(c) (d)
(a) current through r is zero
2 Bwa2
(b) current through r is
5r SectIon 3 (MaxIMuM MarkS : 16)
(c) direction of current in external resistance r is This section contains TWO questions
from centre to circumference Each question contains two columns, Column I and
(d) direction of current in external resistance r is Column II
from circumference to center. Match the entries in Column I with the entries in Column II
16. A glass sphere, refractive index 1.5 and radius 10 cm, One or more entries in Column I may match with one
has a spherical cavity of radius 5 cm concentric with or more entries in Column II
it. A narrow beam of parallel light is directed into 19. An ideal gas undergoes two processes A and B. One
the sphere. Find the final image and its nature. of these is isothermal and the other is adiabatic.

Physics For you | march 16 69


Column-I Column-II (C) Velocity acquired (R) Same if bodies
(A) P (P) Heat supplied by A and B have same
A
during curve A is after travelling diameter,
positive through s would different radius
B be of gyration about
V centre of mass
and different
(B) P (Q) Work done by gas masses
A in both processes
(D) Minimum (S) Different if
positive
B coefficient of bodies have same
V
friction required diameter but
for pure rolling different radius
(C) P (R) Internal energy of gyration about
A
increases in centre of mass
B adiabatic process and different
masses
V (T) Same if they have
(D) P (S) Temperature of same mass and
gas in process B is diameter but
constant different radius
A
B of gyration about
V centre of mass
20. The uniform rigid bodies A and B are allowed to answer keys
roll without slipping on a rough inclined plane.
1. (2) 2. (4) 3. (5) 4. (5) 5. (8)
column-I describes some of the physical quantities
associated with their motions while column-II gives 6. (2) 7. (5) 8. (2) 9. (a, c, d)
you the relation between a physical quantity for 10. (a, b, c) 11. (a, c) 12. (a, c, d)
two different bodies [bodies can be cylinder (solid
13. (a, b) 14. (a, d) 15. (b, d)
or hollow), sphere (solid or hollow), ring or a disc].
Match the entries of column-I with the entries of 16. (a) 17. (a, c) 18. (a, b, c)
column II. 19. A (P, Q); B (R, S); C (P, Q); D (R, S)
Column-I Column-II 20. A (P, Q, S); B (P, Q, S); C (P, Q, S);
(A) Acceleration of (P) Same if A and B D (P, Q, S)
centre of mass of have the same nn
A and B radius of gyration
about centre
of mass, same
diameter but
are of different
materials
Online Test Series
(B) Time taken by (Q) Different if A and Practice Part Syllabus/ Full Syllabus
the two bodies to B are of same Mock Test Papers for

JEE Main
travel a distance diameter and
s on the incline same mass but
would be having different
radius of gyration
about centre of Log on to http://test.pcmbtoday.com
mass

70 Physics For you | march 16

Attempt free online test


Log on to http://test.pcmbtoday.com
Contd. from page no. 30 1 1 3 2
sinC = sin 60 = = =
2 3
22. (a) : Time of free fall of a body from a height h,
The time taken by light to travel some distance in
2h
t= a medium,
g 2
103
Distance from the foot of the tower x
t= = 3 = 3.85 s
2h c 3 108
d = vt = v = 250 m
g 26. (d) : Voltage available across load resistance R
v = 20 5 = 15 V
When velocity =
2
and height of tower = 4h
v 2(4h)
Then, distance x =
2 g
15
2h \ R= = 1.5 103 = 1.5 k
x =v = 250 m 3
10 10
g
27. (a) : Ex and By would generate a plane electromagnetic
d /d
23. (b) : Poisson's ratio, = 0.4 = wave travelling in z-direction. E, B and k form a right
l /l
2 handed system where k is along z-axis. (As, i j = k )
2 d 2 4A
Area, A = r = or d = \ E is along x-axis and B is along y-axis.
4
Differentiating both sides, we get 28. (c) : The equivalent emf of the two cells in parallel
4 circuit,
2d d = A r + r 3 1+ 6 2
eq = 1 2 2 1 = =5V
d r1 + r2 2 +1
2 dd A 2 d d
So, A = ; = =2 The effective internal resistance of two cells in
4 A d / 4 2 d
parallel circuit,
A d rr 2 1 2
Given 100 = 2% \ 100 = 1% req = 1 2 = =
A d r1 + r2 2 + 1 3
d /d The equivalent circuit will be as shown in figure.
Also, = = 0. 4
l /l
d l l 1 d
or = 0.4 100 = 100
d l l 0.4 d
= 2.5 1% = 2.5%

24. (b) : As the current (i) leads the voltage by , it is 5V 15
R-C circuit. 4 Current in ammeter, I = = A
2 32
X 10 +
Hence, tan = C 3
R 29. (c)
1
tan = CwR = 1, 30. (a) : Condition for constructive interference is
4 C wR

As w = 100 rad s1 2t = (2n + 1)
2
1 1
CR = = s where, n = 0, 1, 2, 3, ...
w 100 For minimum thickness, n = 0
Thus, from all the given options, only (b) is correct.
600 109
25. (a) : Given, i = 60 and x = 1 km = 103 m 2 t = t= = = 100 nm
When the total internal reflection just takes place 2 4 4 1. 5
nn
from lateral surface, i = C i.e., C = 60

Physics For you | March 16 71


1. A particle is acted upon by a force of constant 6. The current in a circuit containing a battery
magnitude which is always perpendicular to the connected to 2 W resistance is 0.9 A. When a
velocity of the particle. The motion of the particle resistance of 7 W is connected to the same battery,
takes place in a plane. It follows that the current observed in the circuit is 0.3 A. Then
(a) its velocity is constant the internal resistance of the battery is
(b) its acceleration is constant (a) 0.1 W (b) 0.5 W (c) 1 W (d) 1.5 W
(c) its kinetic energy is constant 7. Two stars of masses m1 and m2 distance r apart
(d) it moves in a straight line. revolve about their centre of mass. The period of
2. The maximum velocity of a particle executing simple revolution is
harmonic motion is v. If the amplitude is doubled r3 r 3 (m1 + m2 )
(a) 2 p (b) 2 p
and the time period of oscillation decreased to 1/3 of 2G(m1 + m2 ) 2G(m1m2 )
its original value, the maximum velocity becomes
(a) 18v (b) 12v 2r 3 r3
(c) 2 p (d) 2 p
(c) 6v (d) 3v G(m1 + m2 ) G(m1 + m2 )
3. A particle moves in x-y plane according to the 8. The arrangement of NAND gates shown below
equations x = 4t2 + 5t + 16 and y = 5t where x, y are effectively works as
in metre and t is in second. The acceleration of the
particle is
(a) 8 m s2 (b) 12 m s2
(c) 14 m s 2
(d) 16 m s2 (a) AND gate (b) OR gate
(c) NAND gate (d) NOR gate
4. In a series LCR circuit, the voltage across the
resistance, capacitance and inductance is 10 V each. 9. Two projectiles A and B thrown with speeds in
If the capacitance is short circuited, the voltage the ratio 1 : 2 acquired the same heights. If A is
across the inductance will be thrown at an angle of 45 with the horizontal, the
angle of projection of B will be
10
(a) 10 V (b) V (a) 0 (b) 60 (c) 30 (d) 45
2
10. A body executes simple harmonic motion. At a
(c) 10 2 V (d) 20 V displacement x, its potential energy is U1. At a
displacement y, its potential energy is U2. What is
5. The angle made by the vector 3 ^i + ^j with x-axis the potential energy of the body at a displacement
(x + y)?
is
(a) U1 + U2 (b) ( U1 + U 2 )2
(a) 30 (b) 45
(c) 60 (d) 90 (c) U 12 + U 22 (d) U1U 2

72 Physics for you | march 16


11. In the system shown in the figure, the acceleration 17. A body is thrown upwards with velocity 40 m s1
of 1 kg block is and it covers 5 m in the last second of its upward
g journey. If the same body is thrown upwards with
(a) downwards velocity 80 m s1, what distance will it travel in the
2
g last second of upward journey ? (Take g = 10 m s2)
(b) upwards (a) 5 m (b) 10 m (c) 15 m (d) 20 m
2
g 18. The magnification produced by an astronomical
(c) downwards
4 telescope for normal adjustment is 10 and the length
g of the telescope is 1.1 m. The magnification, when
(d) upwards
4 the image is formed at least distance of distinct
12. According to Bernoullis equation vision is
1 (a) 6 (b) 14 (c) 16 (d) 18
P + rv 2 + rgh = constant
2 19. A machine gun is mounted on a 2000 kg car on a
where P is pressure, r is density, v is velocity, h horizontal frictionless surface. At some instant, the
is height and g is acceleration due to gravity. The gun fires 10 bullets/second, each of mass 10 g with a
dimensions of the constant are velocity of 500 m s1. The acceleration of the car is
(a) [ML2T2] (b) [MLT2] (a) 0.025 m s2 (b) 0.25 m s2
1 2
(c) [ML T ] (d) [ML2T1] (c) 0.5 m s2 (d) 500 m s2
13. The pressure on the top surface of an aeroplane 20. A light wave and a sound wave have same frequency
wing is 0.8 105 Pa and the pressure on the bottom u and their wavelengths are ll and ls respectively,
surface is 0.75 105 Pa. If the area of each surface is then
50 m2, the dynamic lift on the wing is (a) ll = ls (b) ll > ls
(a) 0.5 104 N (b) 0.25 104 N (c) ll < ls (d) ll = 2ls
4
(c) 5 10 N (d) 25 104 N
21. Two wires of the same material and same length but
14. If pressure of CO2 (real gas) in a container is diameters in the ratio 1 : 2 are stretched by the same
RT a force. The potential energy per unit volume of the
given by P = 2 , then mass of the gas in
2V b 4b two wires will be in the ratio
container is (a) 1 : 1 (b) 4 : 1 (c) 2 : 1 (d) 16 : 1
(a) 11 g (b) 22 g (c) 33 g (d) 44 g
22. In the circuit shown in figure, the current through
15. A coaxial cable consists of two thin cylindrical 2 W resistance is
conducting shells of radii a and b (a < b). The
inductance per unit length of the cable is
m0 (a + b) m0 a
(a) (b) ln
2p a 4 p b
m0 b m0 b
(c) ln (d) ln
4 p a 2 p a (a) zero
25
(b) A
11
16. Which one of the following statements is not correct
regarding a semiconducting material ? 15 10
(c) A (d) A
(a) They have negative temperature coefficient of 11 11
resistance. 23. The radius of gyration of a solid sphere of radius
(b) They have a moderate forbidden energy gap. R about a certain axis is also equal to R. If r is the
(c) Current is carried by electrons and holes distance between the axis and the centre of the
both. sphere, then r is equal to
(d) Every semiconducting material is a tetravalent (a) R (b) 0.5R
element. (c) 0.6R (d) 0.3R

Physics for you | march 16 73


24. In a sample of radioactive substance, what fraction 30. A body of mass 5 kg starts from the origin with an

of the initial nuclei will remain undecayed after half initial velocity u = (30 ^i + 40 ^j) m s 1 . If a constant
of a half-life of the sample? force (6 ^i 5 j^) N acts on the body, the time in
1 1 1 1 which the y-component of the velocity becomes
(a) (b) (c) (d) zero is
2 2 2 4 2 1
(a) 5 s (b) 20 s (c) 40 s (d) 80 s
25. A proton is projected with a velocity 107 m s1 at
right angle to a uniform magnetic field of 100 mT. 31. One mole of gas of specific heat ratio 1.5 being
The time taken by the proton to traverse 90 arc is initially at temperature 290 K is adiabatically
(Mass of proton = 1.6 1027 kg and charge of compressed to increase its pressure 8 times. The
proton = 1.6 1019 C) temperature of the gas after compression will be
(a) 0.05p ms (b) 0.5p ms (a) 580 K (b) 870 K
(c) 5p ms (d) 10p ms (c) 290 2 K (d) 1160 K
26. A cylindrical drum, open at the top, contains 32. The rms value of potential differenceV shown in the
15 litres of water. It drains out through a small figure is
opening at the bottom. 5 litres of water comes out
in time t1, the next 5 litres in further time t2 and the
last 5 litres in further time t3. Then
(a) t1 < t2 < t3 (b) t1 > t2 > t3
(c) t1 = t2 = t3 (d) t1 > t2 = t3 V0 V0 V0
27. When an unpolarized light of intensity I0 is incident (a) (b) V0 (c) (d)
3 2 2
on a polarizing sheet, the intensity of the light which 33. When a proton is released from rest in a room, it
does not get transmitted is starts with an initial acceleration a0 towards west.
1 1 When it is projected towards north with a speed
(a) I0 (b) zero (c) I0 (d) I0
4 2 v0 it moves with an initial acceleration 3a0 towards
28. Three identical charges of magnitude 2 mC are placed west. The electric and magnetic fields in the room are
at the corners of a right angled triangle ABC whose
ma0 3ma0
base BC and height BA are respectively 4 cm and (a) east, down
3 cm. Forces on the charge at the right angled corner e ev0
B due to the charges at A and C are respectively ma0 2ma0
(b) west, up
F1 and F2. The angle between their resultant force e ev0
and F2 is ma0 2ma0
9 1 16 (c) west, down
(a) tan 1 (b) tan e ev0
16 9 ma0 3ma0
(d) east, up
16 16 e ev0
(c) sin 1 (d) cos 1
9 9
34. A proton and an alpha particle are accelerated
29. A linearly polarized electromagnetic wave given through the same potential difference. The ratio of
^
as E = E0 cos(kz wt ) i is incident normally on a the de Broglie wavelengths associated with proton
perfectly reflecting infinite wall at z = a. Assuming and alpha particle respectively is
that the material of the wall is optically inactive, the (a) 1 : 2 2 (b) 2 : 1
reflected wave will be given as (c) 2 2 : 1 (d) 4 : 1
^
(a) Er = E0 cos(kz wt ) i
35. The region surrounding a stationary electric dipole has
^
(b) Er = E0 cos(kz + wt ) i (a) electric field only
(b) magnetic field only
(c) Er = E0 cos(kz + wt ) i^
^ (c) both electric and magnetic fields
(d) Er = E0 sin(kz wt ) i (d) neither electric nor magnetic field

74 Physics for you | march 16


36. A ray of light from a denser medium strikes a rarer 43. Two identical flutes produce fundamental notes
medium at an angle of incidence i. The reflected and of frequency 300 Hz at 27C. If the temperature of
refracted rays make an angle 90 with each other. the air in one of the flutes is increased to 31C, the
The angle of reflection and angle of refraction are number of beats heard per second will be
r and r respectively. The critical angle is (a) 3 (b) 2 (c) 1 (d) 4
(a) sin1(tanr) (b) cos1(tanr) 44. The reddish appearance of rising and setting sun is
(c) sin1(tanr) (d) sec1(tanr) due to
37. A man goes at the top of a smooth inclined plane. (a) reflection of light (b) diffraction of light
He releases a bag to fall freely and himself slides (c) scattering of light (d) interference of light
down on inclined plane to reach the bottom. If u1 45. The equivalent capacitance between the points
and u2 are the respective velocities of the man and X and Y in the circuit with C = 1 mF is
bag at the bottom of inclined plane, then
(a) u1 > u2
(b) u1 < u2
(c) u1 = u2
(d) u1 and u2 cannot be compared
38. A Carnot refrigerator extracts heat from water (a) 0.5 mF (b) 1 mF (c) 2 mF (d) 3 mF
at 0C and rejects it to room at 24.4C. The work
SolutionS
required by the refrigerator for every 1 kg of water
converted into ice is 1. (c) : It is a case of uniform circular motion in which
(Latent heat of ice = 336 kJ kg1) the velocity and acceleration vectors change due to
change in direction. As the magnitude of velocity
(a) 24.4 kJ (b) 30 kJ
(i.e. speed) remains constant, so its kinetic energy
(c) 336 kJ (d) 11.2 kJ
is constant.
39. If wavelength of photon emitted due to transition 2. (c) : If A is the amplitude of simple harmonic
of an electron from the third orbit to the first orbit motion, then
in a hydrogen atom is l, then the wavelength of v = Aw
photon emitted due to transition of electron from 2p
the fourth orbit to the second orbit will be But w = where T is the time period of
T
128 25 36 125 oscillation.
(a) l (b) l (c) l (d) l
27 9 7 11 2p
\ v=A ...(i)
40. A bullet is fired normally towards an immovable T
wooden block. It loses 25% of its kinetic energy When the amplitude is doubled and the time period
in penetrating through a thickness x of the plank. T
decreased to , the maximum velocity becomes
The total thickness penetrated by the bullet into the 3
block is 2p 2p
v = 2A = 6 A = 6v (using (i))
(a) 4x (b) 6x (c) 8x (d) 2x T / 3 T

41. A bar magnet suspended freely in a uniform 3. (a) : As x = 4t2 + 5t + 16 and y = 5t


magnetic field is vibrating with a time period of 3 s. dx d
\ vx = = (4t 2 + 5t + 16) = 8t + 5
If the field strength is increased to 4 times of the dt dt
earlier field strength, the time period will be dy d
vy = = (5t ) = 5
(a) 12 s (b) 6 s (c) 1.5 s (d) 0.75 s dt dt
dv d
42. The momentum of photon whose frequency is u is ax = x = (8t + 5) = 8
dt dt
hu hc u c dvy d
(a) (b) (c) (d) and ay = = (5) = 0
c u hc hu dt dt
Physics for you | march 16 75
The acceleration of the particle is 7. (d) : The situation is shown in figure.
^ ^
a = ax i + ay j = 8 i^ + 0 ^j = 8 i^
or a = 82 = 8 m s 2

4. (b) : As VR = VL = VC = 10 V
\ R = XL = XC and Z = R Let the distances of the stars with masses m1 and m2
and V = IR = 10 V from their centre of mass be r1 and r2 respectively.
When the capacitor is short circuited, the Then
impedance of the circuit is r = r1 + r2 ...(i)
As the necessary centripetal force for their circular
Z = R2 + X L2 = R2 + R2 = 2R motion is provided by the gravitational force
and the current in the circuit is between them, so
V 10 V Gm1m2
I = = = m1r1w2 = m2 r2 w2
Z 2R 2
r m
\ The voltage across the inductance is or m1r1 = m2r2 or r2 = 1 r1
10 V 10 m2
VL = I XL = R= V Substituting this value of r2 in eqn. (i), we get
2R 2
m m r (m + m1 )
^ ^ r = r1 + 1 r1 = r1 1 + 1 = 1 2
5. (a) : Let q be the angle made by 3 i + j with x-axis. m2 m2 m2
^
If i is the unit vector along x-axis, then m2r
or r1 =
^ ^ ^
( 3 i + j ) i m1 + m2
3 3 3
cos q = = = = Gm1m2 m1m2 rw2
^ ^ ^ 2 4 2 \ =
| 3 i + j || i | 3 + 12 12
r2 (m1 + m2 )
3
or q = cos 1 = 30 G(m1 + m2 ) G(m1 + m2 )
2 or w2 = or w =
r 3 r3
6. (b) : Let e and r be the emf The period of revolution is
and internal resistance of
the battery respectively. 2p r3
T= = 2p
Then w G(m1 + m2 )
The current in the circuit is
e
I=
R+r 8. (c) :
In the first case,
I = 0.9 A, R = 2 W The truth table of the given circuit is

\ 0. 9 A =
2W+r
e
...(i)
A B A AB AB (A B) B ( )
Y = AB B
0 0 1 0 1 0 1
In the second case,
0 1 1 1 0 0 1
I = 0.3 A, R = 7 W
1 0 0 0 1 0 1
e
\ 0. 3 A = ...(ii) 1 1 0 0 1 1 0
7W+r
Dividing eqn. (i) by eqn. (ii), we get which is the truth table of NAND gate.
0.9 A 7 W + r Thus the given arrangement of NAND gates works
7W+r
= or 3 = as NAND gate.
0.3 A 2 W + r 2W+r
9. (c) : Let the angle of projection of B be q.
or 6 W + 3r = 7 W + r or 2r = 7 W 6 W = 1 W For projectile A
1W u2 sin2 45
or r = = 0. 5 W Maximum height, HA = A
2 2g

76 Physics for you | march 16


For projectile B 2g g
u2 sin2 q or a= =
Maximum height, HB = B 8 4
2g \ The acceleration of 1 kg block
As both projectiles attained the same heights, g g
= 2a = 2 = upwards
\ HA = HB 4 2
u2A sin245 u2B sin2 q 12. (c) : The dimensions of the constant is equal to the
or = dimensions of each term on the left hand side of the
2g 2g
2
equation. So if we consider the dimensions of P,
sin2 q u2A u then
or = or sin q = A sin2 45
2
sin2 45 u2B uB [constant] = [P]
u 1 [MLT2 ] Force
But A = (given) = 2
[ Pressure = ]
uB 2 [L ] Area

2
2
1 1 1
2
= [ML1T2]
\ sin q = =
2 2 4 13. (d) : The dynamic lift on the wing is
1 1 F = DPA = (P1 P2)A
or sin q = or q = sin 1 = 30 where P1 and P2 are the pressures on the upper and
2 2
bottom surfaces of the wing respectively and A is
10. (b) : In simple harmonic motion, the area of each surface.
Potential energy, U = 1 kx 2 Here, P1 = 0.8 105 Pa, P2 = 0.75 105 Pa, A = 50 m2
1 2 \ F = (0.8 105 Pa 0.75 105 Pa)(50 m2)
\ U1 = kx 2 ...(i) = (0.05 105 Pa)(50 m2)
2
1 = 2.5 105 N = 25 104 N
and U 2 = ky 2
2 ...(ii) 14. (b) : According to van der Waals equation for m moles
At a displacement (x + y), the potential energy of the of real gas
body is
1 1 m2 a
U = k ( x + y )2 = k ( x 2 + y 2 + 2 xy ) P + (V mb) = mRT
2 2 V2
1 2 1 2 1 2
= kx + ky + (2kxy ) or P = mRT m a
2 2 2 V mb
V2
= U1 + U 2 + 2 U1 U 2 (using (i) and (ii)) Comparing it with the given equation
= ( U1 + U 2 )2 RT a
P=
2V b 4b2
11. (b) : If a is the downward 1
acceleration of 4 kg block, we get, m =
2
the upward acceleration of mass of the gas(m)
As m =
1 kg block must be 2a. If T molecular mass(M )of the gas
is the tension in each part 1
\ m = mM = (12 + 32) = 22 g
of string, then 2
The equation of motion of 4 kg 15. (d)
block is 16. (d)
4g 2T = 4a ...(i) 17. (a) : For the body thrown upwards
and the equation of motion of 1 kg block is u = 40 m s1, Dn = 5 m, a = g = 10 m s2
T 1g = 1(2a) ...(ii) a
Multiplying eqn. (ii) by 2, we get As Dn = u + (2n 1)
2
2T 2g = 4a ...(iii) 10
Adding eqns. (i) and (iii), we get \ 5 = 40 (2n 1)
2
2g = 8a On solving, we get n = 4

Physics for you | march 16 77


The body thrown upwards with velocity 40 m s1 21. (d) : The potential energy per unit volume of the wire is
takes 4 seconds to reach the highest point. So the 1 (Stress)2 1 S2
body thrown upwards with velocity 80 m s1 will u= =
2 Young s modulus 2 Y
take 8 seconds to reach the highest point. Hence force
distance travelled in 8th second is As stress, S =
area
10
Dn = 80 (2 8 1) = 5 m S1 F1 A2
2 \ =
Note : A body covers the same distance in the S 2 F A
2 1
last second of its upward journey whatever be its But F1 = F2 (given)
velocity. S1 A2
\ = ...(i)
18. (b) : Let fo and fe be focal lengths of objective and S2 A1
eye piece respectively. As the two wires are of the same material, therefore
For normal adjustment, their Youngs moduli are the same.
f
Magnification of the telescope, m = o i.e., Y1 = Y2
fe
2 2
and length of the telescope, L = fo + fe u1 S1 A2
\ = = (using(i))
u2 S2 A1
Here, m = 10 and L = 1.1 m 2
f d 2 d 4
\ 10 = o or fo = 10 fe ...(i) = 2 = 2 (where, d = diameter)
fe d1 d1

and 1.1 = fo + fe ...(ii)
d1 1
Solving eqns. (i) and (ii), we get But = (given)
fo = 1 m and fe = 0.1 m d2 2
When the image is formed at least distance of 4
u1 2 16
distinct vision D (= 25 cm), then \ = =
u2 1 1
f f 1m 0. 1 m
m = o 1 + e = 1+
fe D 0.1 m 0.25 m
or u1 : u2 = 16 : 1
22. (d) : The distribution of current in various branches
2 7 is shown in figure.
= 10 1 + = 10 = 14
5 5
19. (a) : Here,
Mass of the car, M = 2000 kg
Mass of the bullet, m = 10 g = 10 103 kg
Velocity of the bullet, v = 500 m s1
Number of bullets fired per second, n = 10
As force on the car = rate of change of momentum Applying Kirchhoff s second law to the closed loop
of the bullets ABDA, we get
\ F = nmv = 10 10 103 500 N = 50 N 4I1 + 2(I1 I2) 8I2 = 0
The acceleration of the car is 10 5
F 50 N or 6I1 = 10I2 or I1 = I2 = I2 ...(i)
a= = = 0.025 m s 2 6 3
M 2000 kg Applying Kirchhoff s second law to the closed loop
20. (b) : Let vl and vs be the velocity of light wave and ABCA, we get
the velocity of sound wave respectively. Then 5
4I1 + 8I2 20 = 0 or 4 I2 + 8 I2 = 20 (using (i))
v v l l vl 3
u = l = s or =
ll ls ls vs 44 3 20 15
or I2 = 20 or I2 = = A
But vl > vs \ ll > ls 3 44 11

78 Physics for you | march 16


Physics for you | march 16 79
Substituting this value of I2 in eqn. (i), we get Here, m = 1.6 1027 kg, q = 1.6 1019 C,
5 15 25 B = 100 mT = 100 103 T
I1 = A = A p(1.6 1027 kg )
3 11 11 \ t=
\ The current in 2 W = I1 I2 =
25 15 10
A A= A 2(1.6 1019 C)(100 103 T)
11 11 11 p
= 107 s = 0.5p 107 s
23. (c) : Let M be the mass of the sphere. 2
Moment of inertia of the sphere about a diameter is = 0.05p 106 s = 0.05p ms
2 26. (a) : If h is the initial height of liquid in drum above
I dia = MR2
5
By theorem of parallel axes the small opening, then velocity of efflux, v = 2 gh .
Moment of inertia of the sphere about a certain axis As the water drains out, h decreases, hence v
at a distance r from its centre is decreases. This reduces the rate of drainage of water.
I = Idia + Mr2 = 2 MR2 + Mr 2 Due to it, as the drainage continues, a longer time is
5 required to drain out the same volume of water.
If k is the radius of gyration at that axis, then Thus, t1 < t2 < t3.
I = Mk2 27. (d) : The intensity of light transmitted through the
2
\ Mk 2 = MR2 + Mr 2 I
5 polarizing sheet = 0
2
2 \The intensity of light which does not get transmitted
or k 2 = R2 + r 2
5 I I
= I0 0 = 0
But k = R (given) 2 2
2
\ R2 = R2 + r 2 28. (b) : The situation is shown in figure.
5
2 3 3
or r 2 = R2 R2 = R2 or r = R = 0.6R
5 5 5
24. (a) : The fraction of nuclei which remain undecayed after
time t is
ln 2
lt t
N N 0e ln 2
f = = = e lt = e T1/2 l =
N0 N0 T1/2
T Force on charge at B due to charge at A is
At t = 1/2
2 1 (2 mC)(2 mC)
ln 2 T1/2
ln 2 F1 = along AB

T1/2 2
4 pe0 ( AB)2
f =e =e 2 = e ln 2
and that due to charge at C is
1 1
= = 1 (2 mC)(2 mC)
e ln 2 2 F2 = along CB
4 pe0 (BC )2
25. (a) : When the proton is projected with velocity Let F be the resultant force of F1 and F2.
v at right angle to a uniform magnetic field B, it If F makes an angle q with F2, then
follows a circular path whose radius r is given by 1 (2 mC)(2 mC)
mv r m F 4pe 0 (AB)2
r= or = ...(i) tan q = 1 =
qB v qB F2 1 (2 mC)(2 mC)
where m and q are its mass and charge respectively. 4pe 0 (BC)2
The time taken by the proton to traverse 90 (BC) 2
(4 cm)2 16
= = =
(= p/2) arc is (AB)2 (3 cm)2 9
s (p / 2)r pr pm
t= = = = (using (i)) 16
v v 2v 2qB or q = tan 1
9

80 Physics for you | march 16


29. (b) : Since the wall is perfectly reflecting, amplitude The rms value of V is
(E0) of the linearly polarized electromagnetic wave T T /2 T
remains unchanged.
2
V dt V02 dt + (0) dt T /2
0 0 T /2 V02
Further, as the material of the wall is optically
inactive, there is no phase change (Stokes law). The
Vrms =
T
=
T
=
T dt
0
reflected wave differs from incident wave in only dt
0
one aspect, i.e., it travels alongz axis. Thus,
^ ^ V02 T /2 V02 T V02 V0
Er = E0 cos(kz wt ) i = E0 cos(kz + wt ) i = [t ]0 = = =
T T 2 2 2
^ ^ 1
30. (c) : As u = (30 i + 40 j) m s 33. (c) : In this question, the proton moves from rest
1
\ uy = 40 m s towards west. It is due to a force on the proton by
^ ^ virtue of electric field along west.
and F = (6 i 5 j )N
If m and e are the mass and charge of the proton,
\ Fy = 5 N then the acceleration of proton due to electric field E
The y-component of the acceleration is eE ma0
Fy 5 N = = a0 or E = west
ay = = = 1 m s 2 m e
m 5 kg When the proton is projected towards north with a
Let after time t the y-component of the velocity speed v0, it moves with an acceleration 3a0 towards
become zero. Then west, shows that the proton is experiencing forces
vy = uy + ayt due to electric field along west and magnetic field
or 0 = (40 m s1) + (1 m s2)t or (1 m s2)t = 40 m s1 acting vertically downwards.
40 m s 1 Therefore, the acceleration of proton due to
or t = = 40 s magnetic field
1 m s 2 = 3a0 a0 = 2a0.
31. (a) : If P1 and T1 are the pressure and temperature The force on the proton due to magnetic field
of the gas before compression, P 2 and T 2 are = ev0B = m(2a0)
corresponding quantities after compression, then 2ma0
for an adiabatic compression or B = downwards
ev0
T1 T2 ma0 2ma0
= Thus E = west and B = down
P1( 1) P2( 1) e ev0
( 1) ( 1)/ 34. (c) : de Broglie wavelength associated with a charged
T2 P2 P
or T = P or T2 = T1 2 particle of mass m and charge q accelerated through
1 1 P1
potential difference V is
P2 h
Here, T1 = 290 K, = 8, = 1.5 l=
P1 2mqV
\ T2 = (290 K)(8)(1.51)/1.5) For same V,
= (290 K)(8)0.5/1.5 = (290 K)(8)1/3 1
= 2(290 K) = 580 K l
mq
32. (c) : lp ma qa (4mp )(2e) 8 2 2
\ = = = =
la mp qp (mp )(e) 1 1

or lp : l a = 2 2 : 1
From graph 35. (a) : A stationary charge produces an electric field
T
V = V0 for 0 t only in the space surrounding it.
2
T Thus the region surrounding a stationary electric
V = 0 for tT
2 dipole has electric field only.

Physics for you | march 16 81


36. (a) : The situation is shown in figure. Q2 = mL = (1 kg)(336 kJ kg1) = 336 kJ
The coefficient of performance (a) of a refrigerator is
Q T2 Q (T T )
a= 2 = \ W= 2 1 2
W T1 T2 T2
(336 kJ)(297.4 K 273 K)
or W =
273 K
(336 kJ)(24.4 K)
As the reflected and refracted rays are perpendicular = = 30 kJ
273 K
to each other,
\ r + 90 + r = 180 39. (a)
or r = 180 90 r = 90 r ...(i) 40. (a) : Let the bullet be fired with velocity u. Then
If m1 and m2 are the refractive indices of denser and its initial kinetic energy is
rarer medium respectively, then by Snells law 1
Ki = mu2 (where m is the mass of the bullet)
m1 sini = m2 sinr 2
m On penetrating through a thickness x, the bullet
sin i
or 2 = loses 25% of its kinetic energy. Now its final kinetic
m1 sin r
energy is
But by law of reflection, i = r
Kf = 75% of Ki
m sin r sin r
\ 2= = (using (i)) 75 1 2 3 1 2
m1 sin r sin(90 r ) = mu = mu
100 2 4 2
sin r ...(ii)
= = tan r If f is the resistive force offered by the block to the
cos r
The critical angle C is bullet, then by work-energy theorem
m Ki Kf = fx
sinC = 2 = tan r (using (ii))
m1
or C = sin1(tanr)
EXAM DATES 2016
37. (c) : The gravitational force is a conservative force,
so the work done by it is independent of path. JEE Main : 3rd april (offline),
Hence in both cases 9th & 10th april (online)
1 2 vitEEE : 6th to 17th april
mu = mgh MgiMs : 17th april
2
AMU (Engg.) : 24th april
where h is the height of the inclined plane.
Kerala pEt : 25th & 26th april
or u = 2 gh
Kerala pMt : 27th & 28th april
Clearly, it is independent of mass. ApEAMCEt : 29th april
So, u1 = u2 (Engg. & Med.)
38. (b) : Here, AipMt : 1st may
Mass of water, m = 1 kg COMED K : 8th may
Latent heat of ice, L = 336 kJ kg1 Karnataka CEt : 4th & 5th may
Temperature of hot reservoir (i.e. room), BitsAt : 14th to 28th may
T1 = 24.4C = 24.4 + 273 = 297.4 K wB JEE : 17th may
Temperature of cold reservoir(i.e. water), JEE Advanced : 22nd may
T2 = 0C = 0 + 273 = 273 K AiiMs : 29th may
The amount of heat extracted from water at 0C AMU (Med.) : 1st June
to convert to ice at 0C is JipMER : 5th June

82 Physics for you | march 16


Physics for you | march 16 83
1 2 3 1 2 302 151
or mu mu = fx = = ...(i)
2 4 2 300 150
Since frequency speed of sound
1 1 2
or mu = fx ...(i) u2 v2
4 2 \ =
u1 v1
Let s be the total thickness penetrated by the bullet
into the block. Then by work-energy theorem v2 151
or u2 = u1 = (300 Hz) (using (i))
1 2 v1 150
mu = fs ...(ii)
2 = 302 Hz
Dividing eqn. (i) by eqn. (ii), we get
Hence the number of beats heard per second
1 x
= or s = 4x = u2 u1 = 302 300 = 2
4 s
41. (c) : The time period of vibration of the bar magnet 44. (c) : The reddish appearance of the rising and the
in a uniform magnetic field B is setting sun is due to scattering of light.
I 45. (c) :
T = 2p ...(i)
MB
where I and M are its moment of inertia and
magnetic moment respectively.
When the field strength is increased to 4 times of its
earlier value, the new field strength becomes In the given circuit, capacitor connected between
B = 4B A and B is short circuited and the remaining two
and the new time period becomes capacitors are in parallel.
I I \ The equivalent capacitance between X and Y is
T = 2p = 2p
MB M (4 B ) Ceq = C + C = 2C = 2(1 mF) = 2 mF

1 I T nn
= 2p = (using (i))
2 MB 2 Form IV
1. Place of Publication : New Delhi
But T = 3 s (given) 2. Periodicity of its publication : Monthly
3s 3. Printers and Publishers Name : Mahabir Singh
\ T = = 1.5 s Nationality : Indian
2 Address : Physics For You,
42. (a) : The energy (E) and momentum (p) of photon 406, Taj Apartment,
are related as New Delhi - 110029.
E 4. Editors Name : Anil Ahlawat
E = pc or p = Nationality : Indian
c Address : Physics For You,
hu
But E = hu \ p = 19, National Media
c Centre, Gurgaon
43. (b) : Let v1 and v2 be the speeds of sound at 27C and
Haryana - 122002
at 31C respectively. 5. Name and address of : Mahabir Singh
As v T (in kelvin) individuals who own the 406, Taj Apartment,
newspapers and partners or New Delhi
1/2
v2 T 273 + 31 304 4 shareholders holding more than
\ = 2 = = = 1 +
v1 T1 273 + 27 300 300 one percent of the total capital
I, Mahabir Singh, here by declare that particulars given above are
1 4 true to the best of my knowledge and belief.
=1+ (by binomial theorem)
2 300
Mahabir Singh
Publisher

84 Physics for you | march 16


5. (b) : Note that cohesive force among mercury
molecules is greater than adhesive force between
glass and mercury molecules. Also, adhesive force
solution set-31 between water and glass molecules is greater than
cohesive force among water molecules.
1. (c) : Let at any time t, the displacement of first
particle be S1 and that of second particle be S2. 6. Let the body be in motion for n seconds. If sn is the
distance covered by the body in n seconds, then
1 1
S1 = at 2 and S2 = u t 1
2 a from s = v0t + at 2 ,
1 2
For S2 > S1 sn = an2 (as v0 = 0)
1 1 2ut 2u 2
u t > at 2 t 2 + <0 If snth is the distance covered in the last second of
a 2 a a2
its motion, then
1
a( ) 1
(
u u2 2u < t < u + u2 2u
a ) a
2
a
snth = v0 + (2n 1) = (2n 1)
2
According to the given condition,
Hence, the duration for which particle 2 remains
1 a 1 1
ahead of particle 1 snth = sn or (2n 1) = an2
2 2 2 2
1
( )(
= u + u2 2u u u2 2u
a

) or 2(2n 1) = n2 or n2 4n + 2 = 0

=
2
u(u 2) 4 (4)2 4(1)(2) 4 8
or n= =
a 2 2
2. (b) : According to work energy theorem, (Taking only the positive sign before 8,
Loss in kinetic energy = Work done negative sign implies n < 1s)
1 1 4+ 8
\ 0 mv 2 = Wg + WT or WT = Wg mv 2 n= s = 3.4 s
2 2 2
1 2
= mg 2R mv Also,
2
1 1 1
= 0.1 10 2 0.1 0.1 52 s = an2 = (4)(3.4)2 m 23 m ( a = 4 m s 2 )
2 2 2
= 0.2 0.1 12.5 = 1.45 J 7. Let O be the centre of the hemisphere and OY be
1 2 the axis passing through the vertex of the cone.
3. (a) : For ball A, mg(4h) = mv A 1
2 Volume of cone = r 2h
v A = 8 gh 3
Volume of hemisphere
Similarly, for ball B, 2
1 = r 3
mgh = mv B2 v B = 2 gh 3
2 Densities are same,
Since, the balls collide elastically, masses of the bodies
\ After collision, will be proportional
v A = 2 gh and v B = 8 gh to their respective
volumes.
Ratio of heights attained by ball A and B, i.e.,
If the centre of gravity of the combined mass should
hA v 2A 2 gh 1 lie on O, then,
= = =
hB v B2 8 gh 4 1 2 h 2 3 3r
3 r h 4 + 3 r 8
4. (d) : As L = mvr = constant. 0=
mv 2 m L2 L2 r 2h 2 3
\T = = 2 2 = r 3 + r
r r mr m 3 3

Physics for you | March 16 85


h2 1 20GM 65GM
r 2h2 2 4 3 = r2 \ mv 2 > m +
= r 3 2 8a 8a
3 4 3 8
or h=r 3 45GM 3 5GM
v> i.e., vmin =
8. If the temperature of surrounding increases by DT, 4a 2 a
the new length of rod becomes 10. The frequency in nth mode of vibration of one
l' = l(1 + a DT) end closed and an open organ pipe are given by
Due to change in length, moment of inertia of rod Closed organ pipe,
also changes and is given as
Ml 2 (2n 1)v
I p = c = , where n = 1, 2, 3,
3 4lc
As no external force or torque is acting on rod, thus Open organ pipe,
its angular momentum remains constant during
heating, nv
o = , where n = 1, 2, 3
\ I p w = I p w 2lo
where w is the final angular velocity of rod after Fundamental frequency of closed organ pipe,
heating. v
= 110 Hz
Ml 2 Ml 2 (1 + aDT )2 4lc
or w= w
3 3 330
or w = w(1 2 a DT) lc = m = 0.75 m
4 110
[using binomial expansion for small a]
Thus percentage change in angular velocity of rod 3v
First overtone of closed organ pipe =
due to heating can be given as 4lc
2v
w w First overtone of open organ pipe =
Dw = 100% = 2 a DT 100 % 2lo
w
9. The distance (from the smaller planet) where the These two produce beats of frequency 2.2 Hz
gravitational pulls of the two planets balance each when sounded together, the expressions for beat
other will be given by frequency are
GMm G(16 M )m 3v 2v 2v 3v
= = 2.2 or = 2. 2
x2 (10a x )2 4lc 2lo 2lo 4lc
i.e., x = 2a
330 330
10 a 3 110 = 2.2 or 3 110 = 2.2
lo lo
a B S 2a lo = 1.0067 m or lo
x
M
= 0.9934 m
16 M nn
So the body will reach the smaller planet due to the Solution Senders of Physics Musing
planets gravitational field if it has sufficient energy
Set-31
to cross the point B(x = 2a), i.e.,
1. Subrata Dutta (WB) 2. arnab Jana (WB)
1 2
mv > m(VB VS ) 3. Samrat Gupta (WB) 4. Britant (Bihar)
2
5. asit Srivastava (UP) 6. anurag Mohanty (Odisha)
16GM GM 65GM
Now, VS = + = 8a
7. Manmohan Krishna (Bihar)
2a 10a 2 a Set-30
16GM GM 20GM 1. Gaurav Sharma (New Delhi) 2. Parv Mehta (haryana)
and VB = + =
8a 2a 8a 3. afrid Khan (UP)

86 Physics for you | March 16


Five `hot Jupiter-like planets discovered

S cientists have discovered five new Jupiter-like planets that are similar in characteristics
to our solar systems biggest planet and orbit very close to their host stars.
Researchers from Keele University in UK used the Wide Angle Search for Planets-South
(WASP-South) instrument -an array of eight cameras observing selected regions of
the southern sky, to study five stars showing planet-like transits in their light curve. The
newly discovered planets were designated WASP-119 b, WASP-124 b, WASP-126 b,
WASP-129 b and WASP-133 b. The orbital periods of the planets vary from 2.17 to 5.75
days, and their masses range from 0.3 to 1.2 the mass of Jupiter, with radii between one to
1.5 Jupiter radius, researchers said.
WASP-119 b, which has a mass of 1.2 of the mass of Jupiter, and an orbital period of 2.5
days, is a typical hot Jupiter. Its host star has a similar mass to the Suns but appears to be
much older based on its effective temperature and density .
WASP-124 b, less massive than Jupiter, has orbital period of 3.4 days and a much younger parent star.
WASP-126 b is the lowest-mass world found by researchers. Its low surface gravity and a bright host star make
the planet a good target for transmission spectroscopy. WASP-126b orbits the brightest star of the five. This means
that it can be a target for atmospheric characterisation, deducing the composition and nature of the atmosphere
from detailed study , said Coel Hellier from Keele University .
WASP-129 b, similar in size to Jupiter, has the longest orbital period. WASP-133 b has the shortest orbital period of
the exoplanets detected by researchers.

Moon can affect rainfall, say scientists


T he Moon can affect how heavy the rain is, according to a new study
that could help improve weather forecasts and climate models.
Researchers at Washington University in the US discovered that
Now, well listen to
because the Moon causes the Earths atmosphere to bulge towards it the stars, courtesy
when it is overhead, the air becomes warmer to the extent that it can gravitational
make rain lighter. The change is small, accounting for about 1% of
the total variation in rainfall. Tsubasa Kohyama, a doctoral student, waves
said, This is the first study to connect the tidal force of the moon with
rainfall. Lower humidity is less favourable for precipitation.
T he landmark discovery of
the first direct evidence of
gravitational waves or ripples in
space-time, which Albert Einstein
Nasa craft spots `floating hills in predicted a century ago, will enable
mankind to listen to the stars, and
Plutos heart not just see them, scientists say.
In a breakthrough announcement,
N asas New Horizons spacecraft has captured images of frozen
nitrogen glaciers on Pluto carrying numerous `floating hills that
may be fragments of water ice, giving an insight into the dwarf
scientists from the Laser Interferometer
Gravitational Wave Observatory (LIGO) said that they
planets fascinating and abundant geological activity . have finally detected the elusive gravitational waves, the
ripples in the fabric of space-time.
These hills individually measure one to several
kilometres across, the images show. The hills, Studying gravitational waves will push Einsteins general theory
which are in the vast ice plain named of relativity to its limits, while revolutionising our understanding of
Sputnik Planum within Plutos `heart, the most violent events in the universe, according to researchers
are likely miniature versions of the larger, at Massachusetts Institute of Technology.
jumbled mountains on Sputnik Planums Analysis of the waves suggests they originated from a system
western border. of two black holes, each with the mass of about 30 Suns, that
Since water ice is less dense than nitrogen- gravitationally drew closer to each other. The frequency of these
dominated ice, scientists believe these waves that LIGO is designed to catch are actually in the audible
water ice hills are floating in a sea of frozen range for humans.
nitrogen and move over time like icebergs Accordingly, the signal LIGO received of the black hole merger
in Earths Arctic Ocean. was played on speakers for eager scientists. For this binary black
The hills are likely fragments of the rugged hole system, it made a distinctive, rising whoooop! sound, said
uplands that have broken away and are Matthew Evans, an assistant professor at MIT. This detection
being carried by the nitrogen glaciers into Sputnik means that the stars are no longer silent. Its not that we just look
Planum.`Chains of the drifting hills are formed along the up and see anymore, like we always have we actually can
flow paths of the glaciers. listen to the universe now. Its a whole new sense, Evans said.

Courtesy : The Times of India


Physics for you | march 16 87
Readers can send their responses at editor@mtg.in or post us with complete address by 25th of every month to win exciting prizes.
Winners' name with their valuable feedback will be published in next issue.

across Cut Here

3. A piece of ferromagnetic material that connects two 1 2

or more magnetic cores. (4) 3

4. The product of a component of momentum, and the 4


change in corresponding positional coordinate. (6)
5 6 7 8 9 10 11
5. The ratio of the radiant flux incident on an object to
the flux transmitted. (7) 12

11. A range of frequencies within specified limits used 13

for definite purpose. (4) 14


12. A mechanical device that prevents any sudden or
15
oscillatory motion of a moving part of any piece of
apparatus. (4, 3) 16

14. Kinetic energy released in matter. (5) 17 18 19

15. A mixture of isotopes of an element in proportions 20


that differ from the natural isotopic composition. (7)
21 22
18. A measure of the rate of decay of a periodic quantity.
23
(4, 4)
20. It is an interferometer used to study fine spectrum
lines. (6) 24 25
21. A primary cell, much used before the introduction 26
of accumulators. (6, 4)
23. Roughly spherical ice particles, usually a few
millimeters in radius, produced in very turbulent 8. The vascular layer of the eye lying between the retina
clouds. (4) and the sclera. (7)
24. The absorption of one particle by a system. (7) 9. The distinguishing quality, other than pitch or
25. A unit of power identical to the watt but used for the intensity of a note produced by musical instrument,
reactive power of an alternating current. (3) voice, etc. (6)
26. A line on a chart or graph joining points of equal 10. A small wave. (7)
temperature. (8) 13. A simple machine that converts rotational motion to
down linear motion. (5)
1. The equipotential surface of the gravity potential 16. A permanently electrified substance exhibiting
that coincides with the mean sea level. (5) electric charges of opposite sign at its extremities.
2. A type of alloy containing magnesium (88%), (8)
aluminium (11%) and traces of other elements. (6) 17. The elementary particle that mediates the strong
6. A mixture of free electrons and ions or atomic nuclei. interaction between quarks. (5)
(6) 19. SI unit of magnetic flux. (5)
7. The transfer of matter such as water vapour or heat, 22. A radar-like technique employing pulsed or
through the atmosphere as a result of horizontal continous-wave laser beam for remote sensing. (5)
movement of air. (9) nn

Physics for you | March 16 89


90 Physics for you | March 16

You might also like